2001 -1995

234
The University of Toronto National Biology Competition 2001 Examination Time: 75 minutes Number of Questions: 50 To view the correct answer when you select the "Answer" button you must have Javascript enabled on your web browser (select Options, Network Preferences, and then Languages). When you click on the "Answer..." button, a small window will pop up. You MUST close it, by clicking on "Back To Test". If you do not do this, subsequent windows displaying the answer will not be visible. This document is best viewed with at least Netscape 3.0 or Internet Explorer 3.0 Answer Key General Instructions Do not open this booklet until you are instructed to do so. Print your name at the top of the exam booklet.

Upload: api-3706564

Post on 13-Nov-2014

157 views

Category:

Documents


1 download

TRANSCRIPT

Page 1: 2001 -1995

The University of TorontoNational Biology Competition

2001 Examination

Time: 75 minutesNumber of Questions: 50

To view the correct answer when you select the "Answer" button you must have

Javascript enabled on your web browser (select Options, Network Preferences, and

then Languages).

When you click on the "Answer..." button, a small window will pop up. You MUST

close it, by clicking on "Back To Test". If you do not do this, subsequent windows

displaying the answer will not be visible.

This document is best viewed with at least Netscape 3.0 or Internet Explorer 3.0

Answer Key

General Instructions

Do not open this booklet until you are instructed to do so. Print your name at the top of the exam booklet.

Indicate all of your answers to the questions on the separate Response Form. No credit will be given for anything written in this booklet, but you may use the booklet for notes or rough work. No additional time will be given after the exam to transfer your answers to the Response Form.

After you have decided which of the suggested answers is best, COMPLETELY fill in the corresponding bubble on the Response Form. Give only one answer to each question. If you change an answer, be sure that the previous mark is erased completely.

Use your time effectively. Do not spend too much time on questions that are too difficult. Go on to other questions and come back to the difficult ones later if you have time. It is not expected that everyone will be able to answer all questions.

Page 2: 2001 -1995

Good luck and have fun!

Should you guess the answers to questions about which you are not certain?

Since your score on the exam is based on the number of questions you answered correctly minus one-third of the number you answered incorrectly, it is improbable that guessing will improve your score (it is more likely to lower your score). (No points are deducted or awarded for unanswered questions.) However, if you are not sure of the correct answer but have some knowledge of the question and are able to eliminate one or more of the answer choices, then your chance of getting the right answer is improved, and it may be advantageous to answer such a question.

1. When a cell that was deprived of nutrients was given glucose, the pH within its vacuoles dropped. The best reason for this observation is that:

a. a starved cell pumps protons across the plasma membrane to the cell exterior.b. the dissociation of glucose in the vacuoles releases hydrogen ions.c. protons leave the vacuole by passive transport.d. protons enter the vacuole by passive transport. e. protons enter the vacuole by active transport.

2. Individuals afflicted with haemophilia suffer from excessive bleeding due to the failure of the normal clotting mechanism. The disease is associated with a sex-linked recessive gene. Two brothers are haemophiliacs; their parents do not suffer from excessive bleeding. The probability that their sister inherited the gene for haemophilia is most likely:

a. 0b. 1/4c. 1/2d. 3/4e. 1

Page 3: 2001 -1995

3. If you arranged the units of life listed in the table below into a hierarchy, from smallest (least inclusive) to largest (most inclusive), which unit would be found three levels below a fish?

Organism

Community Molecule

AtomPopulatio

nOrgan

Tissue CellBiosphere

a. Organb. Biospherec. Moleculed. Celle. Population

4. Which of the following best describes Darwin's theory of the mechanism of genetic inheritance?

a. Inheritance of acquired characteristicsb. Blending inheritancec. Molecular inheritanced. Particulate inheritancee. Adaptive inheritance

5. In the absence of oxygen, cells capable of fermentation:

a. accumulate glucose.b. no longer produce ATP.c. accumulate pyruvate.d. oxidize FAD.e. oxidize NADH to produce NAD+.

Page 4: 2001 -1995

6. Which statement is FALSE?

a. Cells that do not divide are usually arrested in the G2 phase.b. Within the centrosome of an animal cell are a pair of centrioles.c. The kinetochore is the point of attachment of the spindle fibre to the chromatid.d. Anaphase begins the instant that the sister chromatids begin to separate.e. A nucleosome consists of part of the DNA molecule wrapped around a group of histone molecules.

7. What is the difference between an acid and a base?

a. An acid undergoes a reversible reaction, while a base does not.b. An acid releases OH- ions in solution, while a base accepts OH- ions.c. An acid releases OH- ions in solution, while a base releases H+ ions.d. An acid releases H+ ions in solution, while a base accepts H+ ions.e. Acidic solutions have higher concentrations of OH- than H+ , while basic solutions have higher concentrations of H+ than OH-.

8. If the inner surface of the ileum in the human small intestine were smooth, rather than being folded and subdivided into villi, which statement would be true?

a. The rate of absorption of digested food molecules would be higher, because the digested food would pass more easily through the digestive tract.b. Digestion would not be as effective, because there would be fewer cells secreting trypsin (a protein-digesting enzyme).c. Humans would have needed to evolve a much longer small intestine to absorb sufficient nutrients from their food.d. Humans would not be able to survive, because the digestive tract would be more susceptible to damage.e. Utilization of cellulose in the diet would not be possible, because the microorganisms that digest cellulose would have nowhere to live.

Page 5: 2001 -1995

9. The total biomass in a terrestrial ecosystem will be greatest for which trophic level?

a. Herbivoresb. Producersc. Primary consumersd. Tertiary consumerse. Secondary consumers

10. After pollination, which of the following events is crucial for fertilization to occur in flowering plants?

a. Sperm swim to the egg and the polar nuclei.b. Petals close around the reproductive parts.c. Meiosis occurs within the pollen grain.d. A pollen tube grows from the stigma to the ovule.e. An insect delivers pollen to the stigma.

11. Plant hormones play a role in regulating seed germination. The graph below shows changes in hormone concentrations (left axis) and hypocotyl growth (right axis) over time for mung bean. Which hormone(s) most likely regulates hypocotyl (bean sprout) growth during mung bean germination?

a. Gibberellic acid aloneb. Auxin alonec. Abscisic acid alone

Page 6: 2001 -1995

d. Both auxin and gibberellic acide. Both auxin and abscisic acid

12. What is the most likely explanation for the observation that two individuals originating from the same clone look different?

a. They developed in different environments.b. They are differentially adapted to the same environment.c. The clone from which they originated had more than one genotype.d. They differ in heterozygosity.e. They differ in homozygosity.

13. In the context of evolution, what is the most important source of new mutations?

a. Exposure to x-rays.b. Exposure to ultra-violet radiation.c. Exposure to chemicals.d. The mispairing of chromosomes during meiosis.e. Errors during DNA replication.

14. Which statement is CORRECT?

a. The vacuole is usually the largest organelle found in plant cells.b. Plants do not have mitochondria.c. Nuclei do not contain proteins.d. The Golgi complex is primarily made up of microtubules.e. The lysosome is only found in prokaryotic cells.

15. In a microbiology laboratory, the technician uses heat to sterilize the nutrient solution that is used to grow a fungus. When the heating system broke down, he sterilized the solution by passing it (in a sterile environment) through a sterile filter with a pore size of 0.2 micrometers. When the fungus was grown on the filtered nutrient solution it stopped growing and looked unhealthy within a

Page 7: 2001 -1995

few days. Which statement is the most likely explanation for the observed effect on the fungus?

a. The nutrient solution contained a virus.b. Heating makes the glucose in the nutrient solution more digestible.c. Filtering removed one of the larger nutrient molecules.d. The nutrient solution contained a bacterium that was pathogenic to the fungus.e. Filtering changed the pH of the nutrient solution.

16. Which factor would contribute the most to increasing the rate of water movement upward in the xylem?

a. The generation of root pressure.b. The availability of soil water.c. The rate of transpiration from the leaves.d. The rate of carbohydrate loading.e. The rate of auxin synthesis.

17. Two alternative alleles, the dominant A and the recessive a, are in equilibrium in a population. The frequency of A is 0.6. The percentage of individuals showing the dominant trait in the population should be:

a. 36%b. 40%c. 60%d. 75%e. 84%

18. Fertile hybrids between different plant species are common in nature and form much of the basis for plant breeding in horticulture and agriculture. The plant species pairs involved in forming fertile hybrids are consistent with:

a. both the biological and the morphological species concepts.b. the biological but not the morphological species concept.c. the morphological but not the biological species concept.

Page 8: 2001 -1995

d. neither the biological nor the morphological species concept.e. all of the species concepts that have been proposed.

19. Which of the following is incorrectly paired with its function?

a. Epididymis: maturation and storage of sperm.b. Prostate gland: secretes testosterone, a male sex hormone.c. Fallopian tube: catches ova and conducts them towards the uterus.d. Seminal vesicles: produce a sugar-containing fluid to nourish sperm.e. Corpus luteum: produces progesterone, a female sex hormone.

20. What was the key evolutionary event that led to the diversification of plants on land? The evolution of:

a. chloroplasts.b. protected seeds.c. tissues to store starch.d. meiosis.e. a water-conducting cell type.

21. Which statement about animal stem cells is FALSE?

a. Stem cells are relatively undifferentiated cells that can divide to produce more differentiated tissue cells.b. Stem cells can be found in tissues that need frequent cell replacement, such as skin, the inner lining of the intestine, and the blood system.c. Dividing stem cells can produce cells that differentiate to replace cells lost to injury and age; for example, stem cells in bone marrow produce red and white blood cells.d. Scientists are able to grow stem cells in the laboratory.e. Recent studies have shown that stem cells from one kind of tissue cannot be made to differentiate into cells of another tissue.

Page 9: 2001 -1995

22. What two characteristics make water different from most other compounds?

a. Its solid state is less dense than its liquid state, and it takes up large amounts of heat to change to its gaseous state.b. Its solid state is less dense than its liquid state, and it takes up only small amounts of heat to change to its gaseous state.c. Its solid state is more dense than its liquid state, and it takes up large amounts of heat to change to its gaseous state.d. Its solid state is more dense than its liquid state, and it takes up only small amounts of heat to change to its gaseous state.e. Its solid state is just as dense as its liquid state, and it takes up no heat to change to its gaseous state.

23. In Drosophila the gene for brown eyes is recessive to its normal allele for red eyes; the gene for curled wings is recessive to its normal allele for straight wings. The two genes show independent assortment. Males heterozygous for both genes are mated to females that are heterozygous for the eye colour gene, but homozygous for the gene for curled wings. What proportion of the offspring should have normal red eyes and normal straight wings?

a. 3/16b. 3/8c. 1/2d. 3/4e. 9/16

24. Some plants contain nitrogen-fixing bacteria (of the genus Rhizobium) in their root nodules. This relationship is known as:

a. an amensalism (one participant harms the other).b. a commensalism (one participant benefits but has no effect on the other).c. a mutualism (both participants benefit).d. interspecific (between species) competition.e. competitive exclusion (competition between species for a limiting resource in which one species completely eliminates the other).

Page 10: 2001 -1995

25. Populations of many species of small mammals – such as lemmings, voles, and snowshoe hares in Canada – undergo large population fluctuations on a predictable basis (e.g., every 3 to 4 years). Which of the following does NOT appear to play a role in these population cycles?

a. Territorial (spacing) behaviour of adult members of these populationsb. Dispersal of individuals away from the area where they were bornc. Stress caused by the effects of living at high densitiesd. High mortality caused by predatorse. Severe weather conditions

26. Natural selection is a process that may result in evolution. If evolution is to occur, which of the following must be true?

i. Individuals within a species are variable.

ii. Some of the variations within species are passed on to the offspring (i.e., they

are inherited).

iii. More individuals are produced than the environment can support; only a

fraction of the offspring produced in each generation survive to reproduce.

iv. The survival and reproduction of individuals are not random; the individuals

who survive and go on to reproduce, or who reproduce the most, are those with

the most favourable variations.

a. i, ii and iiib. i, ii and ivc. i, ii, iii and ivd. iii onlye. iv only

27. Cell division in plants is typically concentrated in meristems. In which of the following regions of the plant would cell division NOT occur?

a. Shoot apexb. Root apexc. Embryod. Woode. Cambium between wood and bark

Page 11: 2001 -1995

28. DNA fingerprinting has become a familiar forensic tool and has been cited in recent criminal trials. It is possible to do DNA fingerprinting with even a very minute sample of DNA because:

a. DNA contains only four different types of nitrogenous bases.b. there are large quantities of DNA in each cell of the body.|c. DNA determines a very specific polypeptide chain.d. one can use the polymerase chain reaction.e. gel electrophoresis is very effective.

29. Which statement about geographical speciation is FALSE? The diverging populations:

a. initially belong to the same species.b. become genetically differentiated from one another.c. acquire some degree of reproductive isolation.d. may respond to natural selection.e. live in the same geographical region.

30. Coupled biochemical reactions are important in determining thermodynamic order within cells. The figure below shows two hypothetical reactions, I and II. Which statement about these reactions is FALSE?

a. The splitting of ATP to ADP + Pi releases about 7.3 kcal/mole.b. The reaction of compound A to produce compound B requires an input of about 4 kcal/mole, which is provided by reaction II.

Page 12: 2001 -1995

c. Reaction I is exergonic, yielding 4 kcal/mole.d. The two coupled reactions release about 3.3 kcal/mole.e. To drive reaction I, the splitting of ATP to ADP + Pi is always required.

31. The content of a large vacuole in a eukaryotic cell has the same osmotic pressure as the surrounding cytoplasm, to prevent it from shrinking or bursting. If this eukaryotic cell is placed in a 3 M sucrose solution:

a. the cell would burst.b. the vacuole would expand and burst.c. the cell and the vacuole would both shrink.d. the cell would shrink but the vacuole would remain the same size.e. nothing would happen, because a 3 M sucrose solution has the same osmotic pressure as the cell cytoplasm.

32. Which statement is CORRECT?

a. Membranes of living organisms are in a fluid state, rather than in a gel state.b. Membranes are semipermeable: large molecules such as proteins can easily move across them.c. Hydrophilic organic molecules easily diffuse across membranes in eukaryotic cells.d. In plants, the plasma membrane is known as the cell wall.e. Rough endoplasmic reticulum is the name given to clusters of ribosomes attached to the cytoskeleton.

33. The sequence of steps in cellular respiration is:

a. Pyruvate oxidation --> electron transport chain citric --> acid (Krebs) cycleb. Pyruvate oxidation --> citric acid (Krebs) cycle --> electron transport chainc. Citric acid (Krebs) --> cycle pyruvate oxidation --> electron transport chaind. Citric acid (Krebs) --> cycle electron transport chain --> pyruvate

Page 13: 2001 -1995

oxidatione. Electron transport chain --> pyruvate oxidation --> citric acid (Krebs) cycle

34. Which statement about the mammalian circulatory system is CORRECT?

a. The average diameter of arteries is greater than that of veins.b. The order of decreasing velocity of blood flow is: arteries > capillaries > veins.c. The total surface area of the capillaries is considerably greater than that of all of the arteries, arterioles, venules, and veins combined.d. In order to return blood to the heart, the blood pressure in veins is higher than it is in the capillaries.e. The electrical activity that coordinates the beating of the heart originates in the atrio-ventricular node.

35. According to the latest estimates of the total number of human genes (based on the findings of the Human Genome Project) there are:

a. less than 20,000 genes.b. 30,000 to 50,000 genes.c. 200,000 to 400,000 genes.d. 500,000 to 1 million genes.e. over 3 million genes.

36. Which statement about the "foreign" genetic material in genetically modified (GM) crop plants or animals is FALSE?

a. It must be inserted again in each generation.b. It alters the phenotype of the GM plant or animal.c. It may be subject to artificial selection by breeders and farmers.d. It may be subject to natural selection by weather, pests, and diseases.e. It may be transferred to related non-crop plants or animals.

Page 14: 2001 -1995

37. Which statement about the two pathways of photosynthesis is FALSE?

a. The first pathway (the light reactions) captures light energy and produces ATP and NADPH + H+.b. The second pathway (the Calvin-Benson cycle) uses the products of the first pathway and CO2 to produce sugars.c. The second pathway is also known as the dark reactions because none of its reactions uses light directly.d. The light reactions occur within chloroplasts; the Calvin-Benson cycle takes place in the cytosol.e. Both pathways stop in the dark.

38. If it was possible to genetically engineer humans to be able to run long distances faster at high altitudes, which of the following would NOT help?

a. Increasing the thickness of the cellular lining of the alveoli.b. Increasing the amount of haemoglobin in each red blood cell.c. Decreasing the binding affinity of oxygen and haemoglobin at very low oxygen partial pressures.d. Decreasing the average size of the alveoli.e. Increasing maximum cardiac output.

39. Which pattern of evolutionary change has NOT been recognized in portions of the fossil record that have been thoroughly sampled?

a. Rapid change from one species to another over a short period of time (punctuation)b. Rapid change from one family or order to another over a short period of time (saltation)c. Stasis (equilibrium)d. Gradual changee. Fluctuating change

40. In the event of global warming which one of the following is most likely to occur?

Page 15: 2001 -1995

a. Existing plant and animal communities will move north in response to warming.b. Agriculture in the Prairie provinces will be redeveloped on soils of the Canadian Shield.c. The anticipated rise in sea level will be caused primarily by the melting of polar ice caps.d. The decomposition of organic matter in the unfrozen surface layer of polar soils will increase.e. Tundra vegetation will increase in area as soils dry out in polar regions.

41. Macromolecules are giant molecules formed by the joining of smaller molecules. The bonds that form between the units of macromolecules are:

a. hydrogen bonds.b. peptide bonds.c. disulfide bonds.d. ionic bonds.e. covalent bonds.

42. Which of the following is NOT a function of the mammalian kidney?

a. Water retentionb. Regulation of sodium in the bloodc. Excretion of toxinsd. Synthesis of ureae. Water excretion

43. In what respects are the photosynthetic adaptations of C4 plants and CAM plants similar?

a. In both cases, the stomata normally close during the day.b. Both types of plants make their sugar without the Calvin-Benson cycle.c. In both cases, an enzyme other than rubisco carries out the first step in carbon fixation.

Page 16: 2001 -1995

d. Both types of plants make most of their sugar in the dark.e. Neither C4 plants nor CAM plants have grana in their chloroplasts.

44. A scientist discovered that a protein associated with the plasma membrane of a cultured animal cell disappeared from the membrane soon after a hormone was added to the cell. After adding the hormone, if she disrupted the cell and centrifuged it, the protein did not stay in solution but went to the bottom of the centrifuge tube. After adding the hormone, if she disrupted the cell and dissolved all membranes with a detergent before centrifuging, the protein remained in solution. Which statement best explains her results?

a. The protein was a peripheral membrane protein that came off the membrane after hormone treatment.b. The hormone caused the cell to make endocytotic vesicles that specifically formed at sites in the plasma membrane that contained the protein.c. The hormone destroyed the protein.d. The hormone bound to the protein to make the protein heavier.e. The hormone caused the plasma membrane to break into fragments, causing the cell to make new plasma membrane without the protein.

45. Which of the following is NOT involved in DNA replication?

a. Helicaseb. RNA primasec. Reverse transcriptased. DNA ligasee. DNA polymerase

46. If an adult human female took a drug that inhibited the release of LH (luteinizing hormone), which of the following would NOT occur?

a. The menstrual cycle.b. Release of an ovum from a mature follicle.

Page 17: 2001 -1995

c. Secretion of FSH (follicle stimulating hormone) from the pituitary.d. Secretion of estrogen by the follicle cells.e. Secretion of GnRH (gonadotropin-releasing hormone) from the hypothalamus.

47. How does a noncompetitive inhibitor inhibit binding of a substrate to an enzyme?

a. It binds to the substrate.b. It binds to the active site.c. It lowers the activation energy.d. It increases the change in free energy ( G) of the reaction.e. It changes the shape of the active site.

48. The figure below shows two nerve action potentials recorded intracellularly in response to stimulating an axon. What experimental manipulation would most likely have caused the change from curve A (solid line) to curve B (dashed line)?

a. A decrease in stimulus intensity.b. A decrease in sodium concentration in the extracellular fluid.c. A decrease in the potassium concentration inside the neuron.d. The presence of a low concentration of tetrodotoxin, a sodium-channel blocker.e. An increase in intracellular calcium concentration.

Page 18: 2001 -1995

49. Thermoregulation in mammals is a balance between heat gain and heat loss. All of the following can affect both heat gain and heat loss EXCEPT:

a. Activity of the sweat glands.b. Thickness of the fat layer under the skin.c. Air movement near the body surface.d. Blood flow in the skin.e. Colour of the body surface.

50. Which statement is FALSE?

a. Nondisjunction refers to the failure of a pair of chromosomes to segregate during meiosis.b. Karyotype refers to the number, forms, and types of chromosomes in a cell.c. Autosomes are cytoplasmic organelles that contain DNA and are capable of self-replication.d. Prophase refers to the onset of nuclear division.e. Centromere refers to the region where sister chromatids join.

The University of Toronto National Biology Competition

2001 Examination

Answer Key

1. e 2. c 3. d 4. b 5. e

6. a 7. d 8. c 9. b 10. d

11. d 12. a 13. e 14. a 15. a

16. c 17. e 18. c 19. b 20. e

21. e 22. a 23. b 24. c 25. e

26. c 27. d 28. d 29. e 30. c

31. c 32. a 33. b 34. c 35. b

Page 19: 2001 -1995

36. a 37. d 38. a 39. b 40. d

41. e 42. d 43. c 44. b 45. c

46. b 47. e 48. c 49. a 50. c

Back to test

On the original test paper (May 2001) there were two correct responses in

each of questions 7 and 38; these have been corrected on the online version of

the test. This was unintentional. If a student selected either of the responses it

was recorded as a correct response.

Question 7 asked "What is the difference between an acid and a base?". Response (d), "An acid releases H+ ions in solution, while a base accepts H+ ions," is the definition (the Brønsted-Lowry definitnion) used most often in biology. Credit was also given to students who selected (e), "An acid releases H+ ions in solution, while a base releases OH ions;" this definition (the Arrhenius definition) is often taught to students in general science classes.

In question 38, both (a) and (d) are correct statements (i.e, they would not help genetically-engineered humans to be able to run long distances faster at high altitudes). To be an incorrect statement (d) should have stated: "Decreasing the average size of the alveoli while keeping their total volume constant."

The University of TorontoNational Biology Competition

2000 Examination

Time: 75 minutesNumber of Questions: 50

NOTE:  

To view the correct answer when you select the "Answer" button you must have Javascript

Page 20: 2001 -1995

enabled on your web browser (select Options, Network Preferences, and then Languages).

When you click on the "Answer..." button, a small window will pop up.  You MUST close

it, by clicking on "Back To Test".  If you do not do this, subsequent windows displaying

the answer will not be visible.

This document is best viewed with at least Netscape 3.0 or Internet Explorer 3.0

Answer Key

General Instructions

Do not open this booklet until you are instructed to do so. Print your name at the top of the exam booklet.

Indicate all of your answers to the questions on the separate Response Form. No credit will be given for anything written in this booklet,  but you may use the booklet for notes or rough work. No additional time will be given after the exam to transfer your answers to the Response Form.

After you have decided which of the suggested answers is best, COMPLETELY fill in the corresponding bubble on the Response Form. Give only one answer to each question. If you change an answer, be sure that the previous mark is erased completely.

Use your time effectively. Do not spend too much time on questions that are too difficult. Go on to other questions and come back to the difficult ones later if you have time. It is not expected that everyone will be able to answer all questions.

Good luck and have fun!

Should you guess the answers to questions about which you are not certain?

Since your score on the exam is based on the number of questions you answered correctly minus one-third of the number you answered incorrectly, it is improbable that guessing will improve your score (it is more likely to lower your score). (No points are deducted or awarded for unanswered questions.) However, if you are not sure of the correct answer but have some knowledge of the question and are able to eliminate one or more of the answer

Page 21: 2001 -1995

choices, then your chance of getting the right answer is improved, and it may be advantageous to answer such a question. 

1. Today there are many different breeds of dogs. What mechanism is responsible for most of this variation?

a. Inbreeding

b. Genetic drift

c. Natural selection

d. Artificial selection

e. Gene flow

2. In the evolution of life on Earth the early primitive cells that were present must have obtained their energy by:

a. glycolysis and fermentation.

b. aerobic respiration.

c. cyclic phosphorylation.

d. noncyclic phosphorylation.

e. oxidative phosphorylation.

3. The excessive use of antibiotics is a concern to the medical community. The concern is that antibiotics will no longer be as effective in treating disease because:

a. humans are evolving a resistance to some antibiotics.

b. viruses are not killed by antibiotics.

c. some bacteria are evolving resistance to antibiotics.

d. antibiotics are very expensive.

e. antibiotics cause mutations.

4. Which of the following "vegetables" is technically a fruit?

a. Potato

b. Lettuce

c. Broccoli

d. Celery

Page 22: 2001 -1995

e. Green bean

5. The concentration of polychlorinated biphenyls (PCB, an organochloride contaminant) in many fish populations has been declining since a ban on their production was instituted in the late 1970s. PCBs remain a potential problem, however, because they are lipophilic and are known to biomagnify. Based on this knowledge, what type of fish is expected to be safest for human consumption (i.e., with the lowest concentration of organochlorides)?

a. Slow-growing fish species.

b. Piscivorous fish species (i.e., which eat other fish).

c. Benthivorous fish species (i.e., which eat invertebrates on the lake bottom).

d. Small (young) fish.

e. Fish species with high fat content.

6. Which statement about chemical bonds is CORRECT?

a. A covalent bond forms between a sodium ion and a chloride ion.

b. A hydrogen bond forms between water molecules.

c. A hydrophobic interaction links an oxygen atom to the hydrogen atoms in a

water molecule.

d. A covalent bond links an iron atom to the protein haemoglobin.

e. An ionic bond binds complementary base pairs together in a double-stranded

DNA molecule.

7. Which of the following statements about atoms are CORRECT?

  i. The atoms which make up an element are usually of the same kind.

  ii. An atom is identified by the number of its protons, which is called the atomic

number.

  iii. Not all atoms of the same element have the same mass number.

  iv. Deuterium and tritium are isotopes of hydrogen; all three have the same

atomic number, 1.

a. i and ii

b. i, ii, iii, and iv

c. i and iii

d. ii and iv

Page 23: 2001 -1995

e. iii and iv

8. Thyroid stimulating hormone (TSH), luteinizing hormone (LH), and oxytocin are all:

a. released from the pituitary gland.

b. tropic hormones (act on other endocrine tissue).

c. steroid hormones.

d. sex hormones.

e. involved in the regulation of blood glucose.

9. Why is the global climate change that is predicted to occur in the next 100 years a concern for ecological communities?

a. Primary production will decline markedly due to decreased atmospheric CO2

concentrations.

b. The tundra of northern polar regions will disappear and be replaced by boreal

forest.

c. Species will not be able to migrate quickly enough to follow the change in

climate.

d. Ocean levels will increase substantially and flood a large portion of present

continents.

e. Ultra-violet (UV) radiation will increase and have a severe impact (e.g.,

deleterious mutations) on communities worldwide.

10. Gas exchange in animals always involves:

a. cellular respiration.

b. breathing movements.

c. active transport of gases.

d. control by the central nervous system.

e. diffusion across membranes.

11. The graph below shows how much nitrate (NO3-) is exported

from the continent towards the ocean by 16 major rivers in the world compared to the density of human populations living in those

Page 24: 2001 -1995

drainage basins (i.e., along these rivers). Nitrate exported through rivers causes eutrophication and has been implicated in the development of toxic algal blooms in marine coastal regions. What conclusion drawn from the graph is CORRECT?

a. Large drainage basins export more NO3-.

b. Small drainage basins export more NO3-.

c. Drainage basins with higher population densities export more NO3-.

d. Drainage basins with lower population densities export more NO3-.

e. All rivers export equally large amounts of NO3-.

12. Which statement about proteins is CORRECT?

a. The primary structure of a protein is composed of many branched chains.

b. Proteins always contain an amino group at one end and the amino acid

methionine at the other end.

c. The formation of disulfide bridges between cysteine amino acids can modify a

protein's primary structure.

d. Charged amino acids are frequently found in the hydrophobic interior of a

folded protein.

e. The quaternary structure of proteins is the result of the interactions of two or

more independent polypeptide chains.

Page 25: 2001 -1995

13. One aspect of genetic engineering involves the modification of proteins to improve their function. Which statement would NOT be a logical goal for researchers in this field?

a. Increase the hydrophobicity of a protein so that it will function in the aqueous

environment of the cell cytoplasm.

b. Improve the binding of a specific substrate to an enzyme.

c. Modification of an enzyme active site to increase the rate of catalysis.

d. Increase the stability of a protein so that it may function at a higher

temperature.

e. Change the amino acid content of the protein to improve its value as a more

nutritious food for animals or humans.

14. A scientist isolated pieces of plasma membrane from an animal cell and put them in an apparatus so that the membrane separated two reservoirs of distilled water. When she added a dilute solution of acetic acid to one side (side A) of the membrane, the pH of the solution on the other side (side B) did not change. She then ground up a culture of animal cells, filtered out the organelles and bits of membrane, and added this cell extract to side A. The pH of the solution in side B then dropped. What is the most reasonable explanation for the drop in pH in side B?

a. The cell extract contained lipids that dissolved in the membrane.

b. The cell extract caused the acetic acid to dissociate.

c. Protons moved across the membrane by diffusion.

d. Protons moved across the membrane by passive transport.

e. Protons moved across the membrane by active transport.

15. Which of the following organelles from a living specimen can NOT be observed under the light microscope (using a x100 objective)?

a. The nucleus of a plant cell.

b. Ribosomes in an animal cell.

c. Chloroplasts in a plant cell.

d. The nucleolus of an animal cell.

e. Mitochondria in a plant cell.

Page 26: 2001 -1995

16. Some single-celled, non-photosynthetic organisms have a water expulsion vacuole that collects water from the cytoplasm and expels it from the cell. If you were trying to grow one of these organisms in the laboratory, which growth medium would be closest to its natural habitat? A dilute, aqueous, nutrient solution:

a. supplemented with 1M NaCl.

b. supplemented with 1M sucrose.

c. supplemented with 1M glucose.

d. supplemented with 1M KCl.

e. with no supplements.

17. Jean Baptiste Lamarck published his theory of evolution in 1809, the year that Charles Darwin was born. Lamarck's theory of evolution has been rejected by modern biologists because:

a. his theory provided a genetic mechanism for how evolutionary change

occurred.

b. his theory was based on special creation.

c. the selection experiments he conducted on giraffe's necks involved artificial

and not natural selection.

d. the characteristics an organism acquires during its lifetime cannot be passed on

to its offspring.

e. his theory was only applicable to humans.

18. Which statement about mitosis and the eukaryotic cell cycle is FALSE?

a. Telophase is defined as the stage at which all the kinetochores arrive at the

equatorial plate and the centromeres begin to divide.

b. Replication of DNA occurs during the S phase of the cell cycle, which is part

of interphase.

c. At the onset of mitosis, the chromatin becomes highly organized as a result of

supercoiling and compacting.

d. Mitosis refers only to the division of the nucleus, but is most often followed by

cytokinesis, the division of the cell.

e. The transition of the cell from the G2 phase to the M phase requires the action

of cyclins.

Page 27: 2001 -1995

19. Which statement about photosynthesis is FALSE?

a. In green plants, both photosystems I and II are required for the synthesis of

NADPH+H+.

b. The first stable product of photosynthesis in the majority of green plants (C3-

type plants) is 3-phosphoglycerate.

c. In normal aerobic environments and saturating levels of light, an increase in

CO2 concentration from 0.03% to 0.06% (300 ppm to 600 ppm) will result in an

increase in the rate of CO2 fixation by most green plants (C3-type plants).

d. Photosynthesis is a redox process: H2O is oxidized, CO2 is reduced.

e. The enzymes required for CO2 fixation in plants are located only in the grana

of the chloroplast.

20. Feather colour in budgies is determined by two different genes that affect the pigmentation of the feathers: Y_B_ is green; yyB_ is blue; and yybb is white. Two blue budgies were paired for life. Over many years, they produced 22 offspring, five of which were white. What are the most likely genotypes for the parents?

a. yyBB and yyBB

b. yyBB and yyBb

c. yyBb and yyBb

d. yyBB and yybb

e. yyBb and yybb

21. A man is brought to court in a paternity case. He has blood type B, Rh positive. The mother has blood group B, Rh negative. The child's blood type is A, Rh negative. Which statement about the man is CORRECT?

a. He is the father.

b. He might be the father.

c. He is not the father.

d. He might not be the father.

e. There is not enough information to make a decision.

Page 28: 2001 -1995

22. A trait in a hypothetical diploid organism is inherited via a single gene with four different alleles. How many different genotypes would be possible in this system?

a. 3

b. 6

c. 8

d. 10

e. 16

23. How are mature human sperm and ova similar?

a. They are approximately the same size.

b. They are formed before birth.

c. They each have a flagellum that provides motility.

d. They are produced from puberty until death.

e. They both have the same number of chromosomes.

24. The frequency of crossing over between any two linked genes is:

a. higher if they are recessive.

b. difficult to predict.

c. determined by their relative dominance.

d. the same as if they were not linked.

e. proportional to the distance between them.

25. Which statement concerning living eukaryotic cells is FALSE?

a. Membranes control which hydrophilic organic molecules pass into or out of

the cell.

b. Lysosomes and centrioles are each bounded by a single membrane.

c. Membranes are usually fluid at 37C.

d. Membranes contain phospholipids and proteins.

e. Membranes are semipermeable.

26. There was great excitement around the world when the sheep "Dolly" was cloned using a nucleus derived from an adult cell of its

Page 29: 2001 -1995

"mother" which was then transplanted into an enucleated egg. There is also excitement when it is announced that genes causing human diseases, like muscular dystrophy, have been cloned. Which statement about these two examples of cloning is CORRECT?

a. They both involve cutting a piece of DNA from the genome.

b. One involves the cloning of a nucleus and the other is the cloning of a piece of

DNA.

c. They both produce products genetically identical to the original donor of

cellular material.

d. They raise no ethical questions.

e. They are similar to deriving a bacterial clone from a single bacterial cell which

divides to form a colony.

27. All fungi:

a. are parasitic.

b. are capable of carrying out photosynthesis.

c. are heterotrophic.

d. live on dead organic matter.

e. are pathogens.

28. On the Galapagos Islands evolutionary divergence has resulted in 14 species of finches that are differentially adapted to feed on seeds, insects, and the buds of various plant species. This example of adaptive radiation occurred because the Galapagos Islands are:

a. close enough to one another to favour considerable inter-island migration.

b. close to the mainland.

c. small, favouring divergence through genetic drift alone.

d. arid and stressful, resulting in many mutations.

e. sufficiently isolated from one another that inter-island migration rarely occurs.

29. NaCl is harmful to most crop plants. A scientist at the University of Toronto genetically modified a plant so that it could be grown in dry parts of the world where the available water has a high level of NaCl. This genetically modified plant copes with the high levels of NaCl by transporting salt into its vacuoles where it accumulates to

Page 30: 2001 -1995

abnormally high levels. Which feature would be observed in the genetically modified plant when compared to a non-modified plant?

a. The leaves in the modified plant are more yellow in colour.

b. The modified plant has salt crystals on the surface of its leaves.

c. The cytosol (the material between the plasma membrane and the vacuole

membrane, excluding the organelles) in the modified plant has a lower osmotic

pressure.

d. The cytosol in the modified plant has a higher osmotic pressure.

e. The osmotic pressure is the same in both plants.

30. Which statement about ATP synthesis is FALSE?

a. ATP is synthesized only in chloroplasts and mitochondria.

b. ATP synthesis in the chloroplast occurs in the thylakoid region of this

organelle.

c. Proton motive force (proton gradient) drives the formation of ATP in

mitochondria.

d. ATP synthases are protein complexes that allow protons to cross membranes.

e. Substrate level phosphorylation of ADP does not require ATP synthase to

catalyse the reaction.

31. Dominant mutations are easier to detect than recessive mutations because they:

a. are always lethal and so their appearance is unmistakable.

b. are expressed in both homozygotes and heterozygotes.

c. occur at a higher frequency.

d. are always neutral in their effect.

e. are always favoured by selection.

32. A man who carries an X-linked allele will pass it on to:

a. all of his daughters.

b. half of his daughters.

c. all of his sons.

d. half of his sons.

e. all of his children.

Page 31: 2001 -1995

33. The main feature of the "biological species concept" is its emphasis on the:

a. large morphological differences between different species.

b. genetic variation within populations.

c. recognition of different species based on their ecological separation.

d. role of sexual reproduction in maintaining diversity within a species.

e. absence of gene flow between different species.

34. Which statement about genetically modified (GM) foods is FALSE?

a. Scientists have used genetic modification, in various forms, as a means of

improving crop yields, crop quality, and pest resistance for many years.

b. Genetic modification includes products made by artificial mutagenesis and by

non-natural crosses between unrelated species.

c. A major difficulty in labelling foods as "GM-free" is that it is virtually

impossible to measure genetically modified DNA or protein molecules in most

foods made from GM crops.

d. The recent decision by McCain Foods to stop processing GM potatoes means

that they will eventually use less pesticides to produce the potatoes that are

required to make fries.

e. A major environmental concern with GM crops is that engineered genes will

escape into the environment, resulting in the origin of "superweeds" (aggressive

undesirable plant species).

35. Of the following taxonomic categories which is the most inclusive (i.e., is the highest in the hierarchy)?

a. Order

b. Subspecies

c. Class

d. Genus

e. Family

36. Some viruses are surrounded by a membranous envelope which originates from the:

Page 32: 2001 -1995

a. plasma membrane of the host cell.

b. synthesis of proteins and lipids inside the virus.

c. synthesis of proteins in the virus and lipids in the host cell.

d. synthesis of lipids in the virus and proteins in the host cell.

e. nucleus of the host cell.

37. In plants, sieve areas are regions of the cell wall with pores that allow cytoplasmic continuity between adjacent:

a. epidermal cells.

b. parenchyma cells.

c. phloem cells.

e. tracheids.

e. xylem vessels.

38. If the haploid number for a species is three, each dividing diploid cell during mitosis will have how many chromatids at anaphase?

a. 3

b. 6

c. 9

d. 12

e. 18

39. A feature of fertilization that is only found in angiosperms is that:

a. the sperm may be carried by wind to the female organ.

b. one sperm fertilizes the egg, while another combines with the polar nuclei.

c. a pollen tube carries a sperm nucleus into the female gametophyte.

d. a chemical attractant guides the sperm towards the egg.

e. the sperm cells have flagella for locomotion.

40. Which statement about mammalian heart function is FALSE?

a. Contraction of the heart originates at the sinoatrial node in the right atrium.

b. The atrioventricular node propagates the contraction to the ventricles.

Page 33: 2001 -1995

c. During atrial contraction venous blood flows into the right ventricle.

d. The pulmonary artery contains oxygenated blood.

e. The left side of the heart pumps only oxygenated arterial blood.

41. The main effect of cytokinins in plants is to:

a. increase the length of internodes on flowering stems.

b. prevent the growth of lateral buds.

c. regulate opening and closing of stomata.

d. stimulate cell division.

e. improve the quality of fruits.

42. In the following list, which term is LEAST related to the others?

a. Duplication

b. Inversion

c. Translocation

d. Nondisjunction

e. Deletion

43. In the central nervous system the amount of voltage change required to open enough sodium channels to initiate a nerve impulse is referred to as the:

a. refractory potential.

b. threshold potential.

c. action potential.

d. polarization potential.

e. resting potential.

44. Which statement about glycolysis is FALSE?

a. During glycolysis, glucose is metabolized to pyruvate with the formation of

ATP and NADH+H+.

b. The initial step in glycolysis requires the phosphorylation of glucose by the

enzyme phosphoglucomutase.

c. The conversion of phosphoenolpyruvate to pyruvate is catalysed by the

Page 34: 2001 -1995

enzyme pyruvate kinase and results in the formation of ATP.

d. The end product of glycolysis, pyruvate, can be used by yeast to produce

ethanol during fermentation.

e. Glycolysis occurs outside the mitochondria.

45. The Swift Fox is a small mammal (about the size of a cat) that once lived throughout the prairie grasslands of central Canada. However, due to a number of factors including hunting, it had completely disappeared from Canada by the late 1970s. In an attempt to restore this species, foxes from American populations have been released in Canada. However, the number of Swift Foxes in Canada still remains low. Which factor is most likely NOT contributing to this?

a. Much of the fox's original grassland habitat has been replaced with agriculture,

which has reduced the quality and quantity of habitat for the foxes and the

availability of suitable prey.

b. The foxes released in early reintroductions did not survive well, likely due to a

lack of knowledge of sources of food and den locations, or because the American

foxes were not adapted to the longer Canadian winters.

c. Coyotes, which are increasing in numbers, are feeding on the foxes, or

competing with the foxes for available resources.

d. Swift foxes are still being trapped for their pelts (fur), as they have been since

the early 1800s.

e. Swift foxes may have been the unintended victim of trapping and poisoning

campaigns directed at other mammals, such as coyotes, wolves, and ground

squirrels.

46. Carbonic anhydrase is an enzyme in red blood cells that catalyses a reaction between carbon dioxide and:

a. bicarbonate.

b. carbonic acid.

c. haemoglobin.

d. oxygen.

e. water.

Page 35: 2001 -1995

47. A certain type of grass has a diploid chromosome number of 8. A similar species of grass has a diploid chromosome number of 10. Interspecific hybridization between the two species results in sterile hybrids that can, nonetheless, reproduce vegetatively. The diploid chromosome number of these hybrids would be:

a. 9

b. 16

c. 18

d. 20

e. 36

48. Various regions of the three embryonic germ layers in vertebrates develop into organs and tissues in the adult. Which match is FALSE?

a. Mesoderm - notochord

b. Endoderm - lungs

c. Ectoderm - liver

d. Mesoderm - muscular system

e. Ectoderm - eye

49. Which characteristic is shared by ammonia, urea, and uric acid?

a. They are all nitrogenous wastes.

b. They all need large amounts of water for excretion.

c. They all require about the same amount of energy to produce.

d. They are all equally toxic.

e. They are all produced in the kidney.

50. You are studying two populations of a species of aquatic insect in two different ponds. Both populations go through several generations each summer. The per capita food supply is the same in both pools throughout the year and it is evenly distributed throughout the water column. You find that the population in pond A is increasing in numbers at a faster rate than the population in pond B. Which observation is most likely NOT contributing to this pattern?

Page 36: 2001 -1995

a. Individuals in pond A have more offspring than individuals in pond B.

b. Individuals in pond A have smaller offspring than individuals in pond B.

c. Individuals in pond A mature at a younger age than individuals in pond B.

d. There is a predator that rarely preys on this species but it is enough of a

threat that it scares this species of insect into hiding in the vegetation at the

very edge of the pond; this predator only lives in pond B.

e. Individuals in pond B have shorter life spans than individuals in pond A.

The University of Toronto National Biology Competition

2000 Examination

Answer Key

1. d 2. a 3. c 4. e 5. d

6. b 7. b 8. a 9. c 10. e

11. c 12. e 13. a 14. e 15. b

16. e 17. d 18. a 19. e 20. c

21. c 22. d 23. e 24. e 25. b

26. b 27. c 28. e 29. d 30. a

31. b 32. a 33. e 34. d 35. c

36. a 37. c 38. d 39. b 40. d

41. d 42. d 43. b 44. b 45. d

46. e 47. a 48. c 49. a 50. b

The University of TorontoNational Biology Competition

1999 Examination

Time: 75 minutesNumber of Questions: 50

Page 37: 2001 -1995

NOTE:  

To view the correct answer when you select the "Answer" button you must have Javascript

enabled on your web browser (select Options, Network Preferences, and then Languages).

When you click on the "Answer..." button, a small window will pop up.  You MUST close

it, by clicking on "Back To Test".  If you do not do this, subsequent windows displaying

the answer will not be visible.

This document is best viewed with at least Netscape 3.0 or Internet Explorer 3.0

Answer Key

General Instructions

Do not open this booklet until you are instructed to do so. Print your name at the top of the exam booklet.

Indicate all of your answers to the questions on the separate Response Form. No credit will be given for anything written in this booklet,  but you may use the booklet for notes or rough work. No additional time will be given after the exam to transfer your answers to the Response Form.

After you have decided which of the suggested answers is best, COMPLETELY fill in the corresponding bubble on the Response Form. Give only one answer to each question. If you change an answer, be sure that the previous mark is erased completely.

Use your time effectively. Do not spend too much time on questions that are too difficult. Go on to other questions and come back to the difficult ones later if you have time. It is not expected that everyone will be able to answer all questions.

Good luck and have fun!

Should you guess the answers to questions about which you are not certain?

Since your score on the exam is based on the number of questions you answered correctly minus one-third of the number you answered incorrectly, it is improbable that guessing will improve your score (it is more likely to lower your score). (No points are deducted or awarded for unanswered questions.) However, if you are not sure of the correct answer but have some knowledge of the

Page 38: 2001 -1995

question and are able to eliminate one or more of the answer choices, then your chance of getting the right answer is improved, and it may be advantageous to answer such a question. 

1. Under the electron microscope, you observe a cell with three different types of large organelles, each bounded by two membranes. The cell is most likely to be from:

a. a plant.

b. an animal.

c. a fungus.

d. a bacterium.

e. a virus.

2. In a biology lab exam, a student is given two cultures of single-celled, colourless organisms. She is told that one culture is of an organism collected from the Pacific Ocean and one is of an organism collected from Lake Winnipeg, but neither culture is identified. A question on the exam asks the student to determine which organism came from which source. Which experiment would best provide the answer to the exam question?

a. Look to see which cells had flagella; the flagellated cells were from Lake

Winnipeg.

b. Put some cells of each culture into a 3 M sucrose solution; the cells that shrank

were from the Pacific Ocean.

c. Put some cells of each culture into a 3 M sucrose solution; the cells that shrank

were from Lake Winnipeg.

d. Put some cells of each culture into distilled water; the cells that burst were

from the Pacific Ocean.

e. Put some cells of each culture into distilled water; the cells that burst were

from Lake Winnipeg.

3. From left to right, identify the class of compounds most likely to contain the functional groups shown below.

Page 39: 2001 -1995

a. Ketones, aldehydes, carboxylic acids

b. Aldehydes, ketones, carboxylic acids

c. Carboxylic acids, ketones, aldehydes

d. Aldehydes, carboxylic acids, ketones

e. Ketones, carboxylic acids, aldehydes

4. Which statement about macromolecules is FALSE?

a. Monomers are the building blocks of polymers.

b. Both DNA and RNA are linear polymers of nucleotides.

c. A monosaccharide is a long-chain polymer formed of simple sugars.

d. Both cellulose and starch are polymers of glucose.

e. Triglycerides (or simple lipids) contain fatty acids and glycerol.

5. The diagram below shows a polypeptide chain and two adjacent cysteine groups. Which statement is FALSE?

a. The sulfhydryl groups on two cysteine side chains in a protein can react to

form a covalent disulfide bridge.

b. When a cysteine residue is not part of a disulfide bridge, the side chain is

hydrophobic.

Page 40: 2001 -1995

c. Loss of hydrogen atoms from sulfhydryl groups of cysteine side chains in a

protein results in a covalent bond that helps stabilize the proteins three-

dimensional structure.

d. The methylene groups on two adjacent cysteine side chains in a protein can

react to form hydrogen bonds that help determine how the protein folds.

e. The cysteine side chains are part of the protein's primary structure, which is

determined by the precise sequence of amino acids in the unbranched

polypeptide chain.

6. A scientist put some photosynthetic plant cells in a solution of inorganic minerals in the dark. He notices that after a few days, the cells could take up mineral A, but not mineral B. Control cells in the same nutrient solution but in the light continued to be able to take up both minerals. Which statement is the best conclusion from all of these observations?

a. Photosynthesis uses light energy to directly transport minerals across the

plasma membrane.

b. Mitochondria do not function in the dark.

c. Mineral A is taken up by diffusion and mineral B by endocytosis.

d. Mineral A is taken up by passive transport and mineral B by active transport.

e. Mineral A is taken up by active transport and mineral B by passive transport.

7. Of the five properties listed below, which are the most important in relation to the function of the plasma membrane in living cells?

i. Selective permeability

ii. Strength

iii. Elasticity

iv. Hydrophilicity

v. Fluidity

a. i and ii

b. i and v

c. ii and iii

d. iii and iv

e. iv and v

Page 41: 2001 -1995

8. The graph below shows the titration of acetic acid (CH3COOH). Which statement is FALSE?

a. At the midpoint, [CH3COOH] = [CH3COO-].

b. At the midpoint, pH is about 4.8.

c. At the endpoint, all of the molecules of acetic acid have been titrated to the

conjugate base, acetate.

d. The buffering range of acetic acid is between about pH 4.2 and pH 6.2.

e. The buffering capacity is most effective at pH values below 4 and above 6.

9. Which statement is FALSE?

a. The Golgi complex forms vesicles that fuse to form the endoplasmic reticulum.

b. Cell walls generally contain high levels of carbohydrate.

c. If a lysosome bursts, its contents can seriously damage the cytoplasm of a cell.

d. Secreted proteins are initially formed by ribosomes attached to the

endoplasmic reticulum.

e. The nucleolus is where ribosomes are assembled.

10. Which of the following is NOT a function of glycolysis?

a. Production of ATP.

Page 42: 2001 -1995

b. Production of NADH.

c. Production of FADH2.

d. Formation of pyruvate.

e. Splitting the carbon skeletons of simple sugars.

11. The chemiosmotic hypothesis attempts to explain:

a. the movement of water into the chloroplast and its use in photosynthesis.

b. the coupling of ATP formation to electron flow in an electron-transport chain.

c. the movement of lipid-soluble molecules through a membrane.

d. the movement of NADH from the cytoplasm into the mitochondrion.

e. the coupling of chemical gradients to osmosis.

12. The graph below shows two hypothetical reactions: reaction course A represents a reaction uncatalysed by an enzyme, whereas B represents an enzyme-catalysed reaction. Which statement is FALSE?

a. When an enzyme changes the activation energy of a reversible reaction of the

type x y, both

the forward and reverse reactions speed up.

b. The uncatalysed reaction (A) requires a higher activation energy than does the

Page 43: 2001 -1995

enzyme-catalysed reaction (B).

c. The change in free energy is the same for both reactions.

d. Adding an enzyme to mediate a reaction (B) does not alter the difference in

free energy between the reactants and products; it only changes the activation

energy and hence the rate constant.

e. Both reactions proceed to equilibrium at the same rate.

13. Which one of the following is an end product of the Calvin cycle, the dark reaction of photosynthesis?

a. ATP

b. NADPH

c. A three-carbon sugar

d. Oxygen

e. Carbon dioxide

14. Some human cells divide approximately every 24 hours. You do some experiments to show that the G1 and S phases take up half of the cycle time. Further microscopic observations demonstrate that the mitotic phase is one hour long. How long does the post-synthesis "gap" take in this case?

a. 1 hour

b. 6 hours

c. 9 hours

d. 11 hours

e. 23 hours

15. If cells in the process of dividing are subjected to colchicine, a drug that interferes with the functioning of the spindle apparatus, at which stage will mitosis be stopped?

a. Anaphase

b. Prophase

c. Telophase

d. Interphase

e. Metaphase

Page 44: 2001 -1995

16. Albinism (lack of skin pigmentation) is caused by a recessive autosomal allele. A man and a woman, both normal pigmented, have an albino child together. The couple then have a second child. What is the probability that the second child will be albino?

a. 0

b. 25%

c. 50%

d. 75%

e. 100%

17. All of the following are used as evidence that genes are on chromosomes EXCEPT:

a. The structure of DNA is a polymer of nitrogenous bases linked together by a

"backbone" of alternating phosphates and sugars.

b. The correlation of the Mendelian law of segregation with the segregation of

chromosomes in meiosis.

c. The correlation of the Mendelian law of independent assortment with the

assortment of chromosomes in miosis.

d. The correlation of Down's syndrome in humans with an extra chromosome 21.

e. The sex linked inheritance of some forms of colour blindness in human males

correlates with the inheritance of one X and one Y chromosome in males.

18. The relationship between genotype and phenotype is rarely simple. The term "pleiotropy" describes:

a. when a heterozygote individual has a phenotype intermediate between those of

the two types of homozygotes.

b. when a heterozygote exhibits phenotypes for both its alleles.

c. the ability of a single gene to affect multiple phenotypic traits.

d. when one gene affects the expression of another gene.

e. characters that vary continuously, indicating an additive effect of two or more

genes on a single phenotypic character.

19. Which statement about meiosis is FALSE?

a. The two meiotic divisions reduce the DNA content per cell to one half of the

initial content.

Page 45: 2001 -1995

b. Crossing over occurs in prophase of the second meiotic division.

c. In many species, the haploid products of meiosis proliferate by mitotic

division.

d. In many multicellular organisms only one of the four products of meiosis is

inherited.

e. There is no DNA replication between the first and second meiotic division.

20. Gene therapy for deleterious mutations in blood cells is becoming possible by introducing normal genes into blood cell precursors in the laboratory and re-injecting these cells into the patient. Which of the following would NOT be associated with this kind of gene therapy research?

a. Attempting to place the normal gene in a vector which will help the gene

integrate into a chromosome.

b. Utilising virus vectors which have had some of their own genes removed.

c. Utilising a suitably modified HIV virus.

d. Preventing transmission of the mutation to the next generation.

e. Finding gene transfer methods for treating other genetic disorders, such as

cystic fibrosis.

21. If 35% of the bases of a DNA molecule are thymine, it follows that it also contains:

a. 30% adenine.

b. 30% cytosine.

c. 15% guanine.

d. 35% uracil.

e. None of the above are correct.

22. Molecular biologists cracked the code of life in the 1960s, when a series of elaborate experiments disclosed the amino acid translations of each of the mRNA codons. How many triplet mRNA codons are there?

a. 16

b. 20

c. 32

Page 46: 2001 -1995

d. 64

e. As many as there are amino acids, plus the initiation and termination codons.

23. You are asked to generate, within two weeks, a collection of recombinant plasmid clones that contain pieces of a large proportion of all human genes. What would be the fastest strategy?

a. Digest human DNA with a restriction enzyme; digest the plasmid with two

further restriction enzymes. Ligate the human and plasmid DNA and transfer

DNA to bacteria.

b. Digest human and plasmid DNA with the same restriction enzyme. Ligate the

human and plasmid DNA and transfer DNA to bacteria.

c. Digest human DNA extensively with a conventional endonuclease; digest the

plasmid with a restriction enzyme producing "blunt ends." Ligate the human and

plasmid DNA and transfer DNA to bacteria.

d. Transform human DNA to bacteria which contain a plasmid. Use a bacterial

recombination system to integrate this DNA into the plasmids.

e. This cannot be done in two weeks or less.

24. The term "translation" refers to:

a. the excision and reintegration of a DNA fragment.

b. a specific type of DNA repair.

c. the synthesis of a polypeptide according to an RNA sequence.

d. the synthesis of RNA according to a DNA sequence.

e. the synthesis of DNA according to an RNA sequence.

25. A population is in genetic equilibrium when genotype and allele frequencies remain the same from one generation to the next. Genetic equilibrium will occur when:

a. populations are small, thus more likely to be affected by genetic drift.

b. beneficial mutations arise.

c. there is no immigration and emigration.

d. there is mating between close relatives.

e. natural selection is acting on a particular phenotype.

Page 47: 2001 -1995

26. In a population of diploid individuals that is in Hardy-Weinberg equilibrium, the frequency of a dominant allele for a certain hereditary trait is 0.3. What percentage of individuals in the next generation would be expected to be homozygous for the dominant trait?

a. 9%

b. 14%

c. 42%

d. 49%

e. 90%

27. When Shakespeare was alive cobs of maize (corn) grew to about 15 cm in length, whereas today they can be almost twice this size. This is because:

a.. of the cooler temperatures and decreased rainfall experienced at that time.

b. agricultural fields in England were small at that time and thus nutrients were

limiting.

c. recombinant DNA technology was not being practised at that time.

d. plant breeding using genetic principles was not introduced until the 20th

century.

e. of the lack of genetic variation in maize from Mexico, where maize originated.

28. Many people were sceptical of the theory of evolution when Darwin first proposed it. Darwin received such sharp criticism because:

a. the bones in the wings of bats, fins of porpoises, and legs of humans were

known to be analogous structures.

b. he could not explain completely how evolution occurred because he did not

know the mechanism of inheritance.

c. the fossil record indicated that there were links between birds and reptiles.

d. Earth was thought to be much older than it actually is.

e. he proposed that chimpanzees had evolved into humans.

29. The sequence of events in geographic speciation is most likely to be:

Page 48: 2001 -1995

a. Genetic divergence --> geographic barrier --> reproductive isolation.

b. Geographic barrier --> genetic divergence --> reproductive isolation.

c. Reproductive isolation --> genetic divergence --> geographic barrier.

d. Geographic barrier --> reproductive isolation --> genetic divergence.

e. Genetic divergence --> reproductive isolation --> geographic barrier.

30. Elderly people are advised to get influenza (flu) vaccinations every year. Each year, a different type of flu vaccine has to be made. This is because:

a. different viruses attack people of different ages, so each year as the population

ages, a new vaccine must be produced.

b. antibodies to the flu vaccine do not survive very long in the blood.

c. vaccines are unstable and cannot be stored for more than one year.

d. the body learns to destroy the antibodies made against the vaccine, so a new

type of vaccine is needed for each vaccination.

e. flu viruses change their genetic constituents so rapidly that vaccines against

them rapidly become obsolete.

31. Which important aspect of the classification of all organisms is attributed to Carl Linnaeus?

a. The use of Latin.

b. The use of branching diagrams (trees) to depict relationships among groups.

c. The use of two-part names (binomials).

d. The use of standardized common names.

e. The use of keys for identification.

32. It is generally agreed that prokaryotes constituted the first life on Earth. It is also generally accepted that the early eukaryotes were:

a. photosynthetic.

b. chemosynthetic.

c. heterotrophs.

d. multicellular.

e. unicellular.

Page 49: 2001 -1995

33. Which group of organisms has the largest number of described species?

a. Arthropods

b. Vertebrates

c. Bacteria

d. Fungi

e. Flowering plants

34. A mycorrhiza is a mutualistic association between a fungus and a plant's roots. Which statement best characterizes this association?

a. The fungus provides nothing to the plant; the plant provides food to the

fungus.

b. The fungus secretes toxins to kill the plant; the plant provides minerals to the

fungus.

c. The fungus provides minerals to the plant; the plant secretes toxins to kill the

fungus.

d. The fungus provides minerals to the plant; the plant provides food to the

fungus.

e. The fungus provides minerals to the plant; the plant provides nothing to the

fungus.

35. Which statement about water movement in plants is FALSE?

a. It occurs from a region of low to high concentration.

b. It occurs in dead cells, called tracheids and vessel elements.

c. It depends on the cohesive properties of molecules.

d. It depends on the adhesive properties of molecules.

e. Water moves from the leaf to the atmosphere by diffusion.

36. It is common practice for farmers to pick tomatoes while they are green and to store them in a warehouse prior to shipment to the local grocery store. Prior to shipment, the tomatoes are treated with a growth regulator to induce ripening. Which growth regulator is used?

a. Cytokinin

b. Auxin

Page 50: 2001 -1995

c. Abscisic acid

d. Gibberellin

e. Ethylene

37. In which region of the vertebrate digestive tract does the majority of nutrient absorption into the blood stream occur?

a. Stomach

b. Colon

c. Small intestine

d. Large intestine

e. Esophagus

38. The separation of oxygen from hemoglobin is enhanced near exercising muscle (this is known as the Bohr effect) and is caused by:

a. oxygen binding to hemoglobin in the lung.

b. carbon monoxide binding to oxygenated hemoglobin.

c. carbonic anhydrase.

d. the interaction of lowered pH with oxygenated hemoglobin.

e. the interaction of carbon dioxide with oxygenated hemoglobin.

39. Beginning from the anterior (= superior) vena cava, what is the correct pathway of blood through the mammalian heart? (R = right, L = left)

a. R atrium --> R ventricle --> pulmonary vein --> lung circuit --> pulmonary

artery --> L atrium --> L ventricle --> aorta.

b. L atrium --> L ventricle --> pulmonary artery --> lung circuit --> pulmonary

vein --> R atrium --> R ventricle --> aorta.

c. Aorta --> L atrium --> L ventricle --> pulmonary vein --> lung circuit -->

pulmonary artery --> R ventricle --> R atrium.

d. R ventricle --> R atrium --> pulmonary artery --> lung circuit --> pulmonary

vein --> L atrium --> L ventricle --> aorta.

e. R atrium --> R ventricle --> pulmonary artery --> lung circuit --> pulmonary

vein --> L atrium --> L ventricle --> aorta.

Page 51: 2001 -1995

40. Which of the following is a "key event" in the evolution of vertebrates?

a. Gut formation from the coelom

b. Formation of the neural crest

c. Spiral cleavage

d. Gastrulation following cell cleavage

e. Organogenesis to produce organs

41. Oogenesis is the process in the ovary that results in the production of female gametes. Which statement about oogenesis in humans is FALSE?

a. Oogenesis begins before birth.

b. In oogenesis, unequal cytokinesis results in one single large daughter cell,

which goes on to form the egg.

c. At birth, an ovary already contains all the cells it will ever have that will

develop into eggs.

d. The process of oogenesis is completed when the egg cell is penetrated by

sperm.

e. Oogenesis continues throughout the life of the individual.

     [see comments]

42. As a nerve impulse passes along an axon:

a. the membrane potential changes from positive to negative and then back again.

b. sodium ions flow out through ion channels and potassium ions flow in.

c. sodium channels open as the membrane potential becomes less negative.

d. the sodium-potassium pump moves sodium ions into the cell.

e. potassium channels close as the membrane potential becomes more positive.

43. A main function of the autonomic nervous system, which consists of the sympathetic and parasympathetic divisions, is to:

a. act as an inhibitory system for skeletal muscle.

b. control the activity of a variety of secretory cells throughout the body.

c. communicate between the two halves of the brain.

d. control involuntary reflexes such as the knee-jerk response.

Page 52: 2001 -1995

e. initiate the heart beat.

44. Some athletes take "steroids" in an attempt to enhance their physical performance. This can lead to decreased sperm production and even sterility. What is the most likely explanation for this effect?

a. Interference in the proper negative feedback control of testosterone by

luteinizing hormone.

b. Increased stimulation of the anterior pituitary to produce luteinizing hormone.

c. Excessive growth of testicular tissue.

d. Suppression of the natural production of thyroxin due to the destruction of

thyroid tissue.

e. Excessive diversion of protein and other metabolites from the gonads to

muscle tissue.

45. If you were outside for a long time on a hot, dry day, without anything to drink, which of the following would happen?

a. The production of thyroxin by your thyroid gland would increase.

b. The osmotic pressure of your blood would decrease.

c. The re-absorption of fluids from your kidney tubules would decrease.

d. The concentration of urea in your urine would decrease.

e. The secretion of anti-diuretic hormone from your pituitary gland would

increase.

46. In which ecosystem would you find the highest biodiversity?

a. Freshwater marsh

b. Prairie grassland

c. Tundra

d. Boreal forest

e. Cultivated field

47. The diagram below shows the feeding interactions between nine species (A to I) of a food web with four trophic levels. Which statement about this food web is CORRECT?

Page 53: 2001 -1995

a. Species A is a herbivore.

b. Species D is a carnivore.

c. Species G is an omnivore.

d. Species H is a predator.

e. Species I is a primary producer.

48. What is the "carrying capacity" of a population?

a. The rate at which the density of individuals increases over time.

b. The maximum number of individuals which can be supported in a given

environment.

c. The proportion of individuals which are most responsible for population

growth.

d. The minimum number of individuals necessary to avoid extinction of the

population.

e. The number of individuals available to support higher trophic levels.

49. Which of the following is NOT a major cause of species extinction today?

a.. Introduction of diseases

b. Global climate change

c. Hunting

d. Habitat destruction

e. Introduction of predators

50. An experiment was conducted to determine the feeding preference of guppies. Four treatments were conducted where

Page 54: 2001 -1995

guppies were offered different mixtures of worms and flies: from a mixture of 20% worms (and 80% flies) to a mixture of 80% worms (and 20% flies). The stomach contents of the guppies were compared to determine what they actually ingested. The results are shown below (circles = mean values, vertical bars = total ranges). What can be concluded from these results?

a. Guppies prefer worms.

b. Guppies prefer flies.

c. Guppies prefer the least abundant food available.

d. Guppies prefer the most abundant food available.

e. Guppies do not have a feeding preference.

The University of Toronto National Biology Competition

1999 Examination

Answer Key

1. a 2. d 3. b 4. c 5. d

6. d 7. b 8. e 9. a 10. c

Page 55: 2001 -1995

11. b 12. e 13. c 14. d 15. e

16. b 17. a 18. c 19. b 20. d

21. c 22. d 23. b 24. c 25. c

26. a 27. d 28. b 29. b 30. e

31. c 32. e 33. a 34. d 35. a

36. e 37. c 38. d 39. e 40. b

41. e 42. c 43. b 44. a 45. e

46. a 47. d 48. b 49. b 50. d

Note about question 41

The University of Toronto National Biology Competition

1998 Examination Time: 90 minutes

Number of Questions: 60

NOTE:  

To view the correct answer when you select the "Answer" button you must have Javascript enabled on your Web browser (select Options, Network Preferences, and then Languages).

When you click on the "Answer..." button, a window will pop up. You MUST close it, by clicking on "Back To Test". If you do not do this, subsequent windows displaying the answer will not be visible.

This document is best viewed with Netscape 3.0 or Internet Explorer 3.0.

Answer Key

General Instructions

Page 56: 2001 -1995

Do not open this booklet until you are instructed to do so.

Print your name at the top of this booklet.

Indicate all of your answers to the questions on the separate Response Form.  No credit

will be given for anything written in this booklet, but you may use the booklet for notes or

rough work.  No additional time will be given after the exam to transfer your answers to

the Response Form.

After you have decided which of the suggested answers is best, COMPLETELY fill in the

corresponding bubble on the Response Form. Give only one answer to each question.  If

you change an answer, be sure that the previous mark is erased completely.

Use your time effectively.  Do not spend too much time on questions that are too difficult.

Go on to other questions and come back to the difficult ones later if you have time.  It is

not expected that everyone will be able to answer all questions.

Good luck and have fun!

Should you guess the answers to questions about

which you are not certain?

Since your score on the exam is based on the number of questions you answered correctly minus one-third of the number you answered incorrectly, it is improbable that guessing will improve your score (it is more likely to lower your score). (No points are deducted or awarded for unanswered questions.) However, if you are not sure of the correct answer but have some knowledge of the question and are able to eliminate one or more of the answer choices, then your chance of getting the right answer is improved, and it may be advantageous to answer such a question.

1. Which statement best describes the process of endocytosis?

a. A vesicle within a cell fuses with the plasma membrane and releases its

contents to the outside.

b. Solid particles or liquids are taken up by a cell through invagination of the

plasma membrane.

c. Investment in one cytosis reduces the ability of the parent to assist another

cytosis.

d. One region of an embryo directs the development of a neighbouring region of

an embryo through movement of cells.

e. An organism obtains its energy from light and organic compounds.

2. The primary role of a lysosome is:

Page 57: 2001 -1995

a. intracellular digestion.

b. ATP synthesis.

c. lipid transport.

d. carbohydrate storage.

e. protein synthesis.

3. Which statement about chloroplasts is FALSE?

a. They are organelles with a double membrane.

b. They contain their own genetic information and ribosomes.

c. They are found in eukaryotic and prokaryotic cells.

d. The thylakoid membranes within the chloroplast contain chlorophyll.

e. They contain ATP.

4. The graphs below show the relationship between the rate of water loss by transpiration and leaf

temperature. Based on these results, which plant is best adapted for conserving water in a hot dry

environment?

a. Sunflower

b. Maple

c. Creosote

d. Grass

e. Pine

5. Which statement about microtubules is FALSE?

a. They form the spindle fibres involved in movement of chromosomes during

mitosis and meiosis.

Page 58: 2001 -1995

b. They are involved in cell support and shape.

c. They are involved in the movement of flagella.

d. They are involved in the movement of cilia.

e. They are made up of carbohydrate molecules called phospholipids.

6. Which structure is present in a bean sprout (young seedling), but not in a bean seed?

a. Elongated hypocotyl

b. Radicle

c. Two cotyledons

d. Endosperm

e. Micropyle

7. Plant hormones play a role in regulating the development of plant seeds. The graphs below plot

embryo growth and the changes in hormone concentration over time. Based on these results,

which hormone(s) most likely regulate(s) embryo growth?

a. Auxin alone

Page 59: 2001 -1995

b. Gibberellic acid alone

c. Abscisic acid alone

d. Both auxin and gibberellic acid

e. Both auxin and abscisic acid

8. Cell division typically occurs only in the meristematic regions of plants. In which region would

cell division NOT occur?

a. Shoot apex

b. Wood

c. Root apex

d. Cambium between wood and bark

e. Expanding leaf

9. How many times does cytokinesis occur during the entire process of meiosis?

a. 1

b. 2

c. 3

d. 4

e. 5

10. In humans, the sex ratio is very close to 50:50. The best genetic explanation for this is:

a. crossing over.

b. independent assortment.

c. linkage.

d. transformation.

e. segregation.

11. There are three alleles at the ABO antigen locus in humans. The A and B alleles produce A and

B antigens, respectively, while the O allele produces no antigen. For which of the following

phenotypes could you be certain that an individual (with that phenotype) is a homozygote?

a. Type A only

b. Type B only

c. Type O only

d. Types A, B, and O

e. None of the phenotypes

Page 60: 2001 -1995

12. Quebec premier Lucien Bouchard had his leg amputated because of an infection known as the

"flesh-eating disease." This disease seems to have become a problem in Canada because of:

a. recent contact between humans and rodents.

b. travel between Africa and Canada.

c. the transmission of a virus from chickens to humans.

d. the evolution of antibiotic resistance.

e. contaminated hamburgers at fast food chains.

13. Which of the following would NOT be found in the cell of a bacterium?

a. DNA

b. Ribosomes

c. Plasma membrane

d. Pili

e. Mitochondria

14. Which statement is FALSE?

a. Bacteria are necessary for decomposition.

b. Bacteria can cause diseases of plants.

c. Bacteria are part of your "personal" microflora.

d. Bacteria are necessary for biological nitrogen fixation.

e. Bacteria are necessary for photosynthesis.

15. In Canada, recent outbreaks of meningitis, caused by a virus, were controlled by immunization

with a vaccine. Unfortunately, AIDS caused by the HIV virus has not yet been controlled by a

vaccine because of the:

a. cost of development and production of vaccines.

b. availability of effective antibiotics.

c. high degree of variability in the HIV virus.

d. complex structure of the HIV virus.

e. lack of antigenic proteins on the HIV virus.

16. Which statement is CORRECT?

a. Appendicitis can lead to the condition known as peritonitis, in which parts of

the liver are destroyed.

Page 61: 2001 -1995

b. Heartburn is caused by the acidic contents of the stomach escaping into the

thoracic cavity.

c. Hepatitis is a condition where there are insufficient bile salts to dissolve

cholesterol; the cholesterol then forms large crystals.

d. Hiatal hernia is a condition in which part of the stomach may protrude through

the diaphragm into the thoracic cavity.

e. Gallstones are mainly composed of uric acid and can often be treated with

ultrasound.

17. In herbivorous mammals, the cellulose of plant cell walls:

a. is digested by enzymes produced by symbiotic microorganisms.

b. is digested by cellulase, which is secreted by the lining of the rumen.

c. is hydrolysed to simple molecules by large quantities of saliva.

d. cannot be digested and therefore forms the bulk of the faeces.

e. cannot be digested, but the cell walls are broken down by the mechanical

action of the teeth so that the cell contents are released and become available.

18. Which statement about gas exchange in vertebrates is FALSE?

a. Most of the carbon dioxide in the blood is carried in the form of bicarbonate

ions.

b. Without diffusion, there would be no gas exchange.

c. If the concentration of carbon dioxide in the alveolar air increases, the rate of

carbon dioxide exchange decreases.

d. Less carbon dioxide would be carried by the blood if there were fewer red

blood cells.

e. An increase in oxygen concentration would result in less oxygen binding to the

iron atoms in haemoglobin.

19. The trachea, bronchi, and bronchioles of humans have all of the following functions

EXCEPT:

a. increasing the surface area available for gas exchange.

b. moistening the incoming air.

c. conducting mucous away from the alveoli.

d. warming the incoming air to body temperature.

e. conducting air from the exterior of the body to where the respiratory gases can

enter or leave the blood.

Page 62: 2001 -1995

20. Which statement is FALSE?

a. Veins are typically larger in diameter than arteries.

b. Because of their small size, capillaries contain blood that is moving more

quickly than in other parts of the circulatory system.

c. The walls of arteries are elastic, enabling them to stretch and shrink with

changes in blood pressure.

d. Veins contain more blood than any other part of the circulatory system.

e. The blood pressure in the veins is normally too low for blood to return to the

heart without the action of skeletal muscles.

21. Some of the important events in the human female reproductive cycle are listed below. Which

of the following best describes their sequence?

1 - A drop in progesterone levels

2 - Secretion of follicle stimulating hormone

3 - Growth of the corpus luteum

4 - Oogenesis

5 - Menstruation

6 - Ovulation

7 - Growth of the follicle

8 - A sudden increase in levels of luteinizing hormone

a. 2 --> 4 --> 7 --> 8 --> 6 --> 3 --> 1 --> 5b. 3 --> 8 --> 4 --> 7 --> 1 --> 6 --> 5 --> 2c. 5 --> 7 --> 6 --> 2 --> 8 --> 3 --> 1 --> 4d. 6 --> 3 --> 1 --> 5 --> 8 --> 7 --> 2 --> 4 e. 8 --> 3 --> 6 --> 4 --> 2 --> 7 --> 1 --> 5

22. The mammalian heart beat:

a. stops when the nerve supply to the heart is cut.

b. originates at the atrio-ventricular node.

c. decreases when stretch receptors in the vena cava are stimulated.

d. slows down when activity in the vagus nerve increases.

e. increases when carbon dioxide levels in the blood decrease.

23. The embryo of which of the following organisms does NOT have an amnion?

a. A bird

b. A fish

Page 63: 2001 -1995

c. An alligator

d. A lizard

e. A human

24. Which of the series of atoms listed below are ranked (from left to right) in terms of increasing

electronegativity?

a. F, N, C, H, Na

b. Na, H, C, N, F

c. Na, C, H, N, F

d. Na, H, C, F, N

e. F, N, H, Na, C

25. __________ bonds are important components that hold DNA strands together in the DNA

double helix. (fill in the blank)

a. Ionic

b. Covalent

c. Polar covalent

d. Peptide

e. Hydrogen

26. Which statement about amino acids is CORRECT?

a. They always have at least one amino group and at least one carboxyl group.

b. In the formation of proteins, it is a condensation reaction that links the amino

group of one amino acid to the variable side chain of the adjacent amino acid.

c. The variable side chains of all of the amino acids are highly reactive and carry

a charge at neutral pH.

d. The peptide bond that links amino acids together in a protein is a type of ionic

bond, which explains why proteins are unstable at high temperatures.

e. The variable side chains of all amino acids are hydrophillic, which allows

polypeptides to remain soluble in an aqueous environment.

27. Which statement about nucleotides is CORRECT? Nucleotides:

a. such as ATP are used in the in vivo synthesis of nucleic acids.

b. are composed of only a pentose sugar and a nitrogenous purine or pyrimidine

base.

c. such as GTP replace ATP in the synthesis of RNA molecules.

Page 64: 2001 -1995

d. contain a deoxyribose sugar if they are components of RNA, and a ribose

sugar if they are components of DNA.

e. may contain a purine nitrogenous base, which is a single ring structure.

28. Which statement about proteins is CORRECT?

a. Protein quaternary structure is determined solely by the primary amino acid

sequence.

b. Protein sequences that span a lipid bilayer membrane usually contain a number

of charged amino acids.

c. Examples of protein secondary structure include an alpha helix or a beta

pleated sheet.

d. Protein tertiary structure is the result of the interaction of two or more

independent polypeptide chains.

e. Protein primary structure is altered by the interaction of the polypeptide chain

with co-factors required for enzyme activity, such as metal ions or vitamins.

29. The protein complexes that constitute a cell's oxidative metabolism in the mitochondrion are:

a. dissolved within the matrix fluid.

b. in the outer compartment between the two membranes.

c. on the cytoplasmic face of the outer membrane.

d. dissolved in the fluid of the outer compartment.

e. embedded within the inner membrane.

30. The initial step in the biosynthesis of ATP by chemiosmosis in the mitochondrion is the:

a. accumulation of sufficient inorganic phosphate by active transport.

b. removal of phosphate from glucose-6-phosphate.

c. pumping of protons into the outer compartment.

d. pumping of electrons into the matrix.

e. diffusion of protons into the outer compartment.

31. You are a membrane biologist and have just elucidated the molecular structure of a protein that

penetrates the thickness of the plasma lipid bilayer with several non-polar, parallel helical

segments. The next most logical set of experiments you conduct should be designed to

characterize:

a. a specific cell surface marker.

b. a channel protein.

Page 65: 2001 -1995

c. a microtubule extension.

d. a receptor.

e. a highly charged segment of the protein on the side protruding within the cell.

32. If one mole of glucose and six moles of oxygen yield six moles of carbon dioxide and six

moles of water when completely metabolized in a living cell, the net energy yield should be:

a. two ATP molecules.

b. two NADH and two ATP molecules.

c. six NADH and two FADH2 molecules.

d. 686 kilocalories.

e. 686 kilocalories for each of the two turns of the Krebs (citric acid) cycle.

33. Which statement is FALSE?

a. NADH dehydrogenase is the terminal electron acceptor of the electron

transport chain.

b. O2 + 4H+ + 4e-  --> 2H2O

c. The electron transport chain uses electrons from the oxidation of glucose to

drive proton pumps.

d. The ultimate acceptor of electrons harvested from pyruvate is oxygen gas

which is reduced to form water.

e. The oxidation of glucose to two pyruvate molecules from glycolysis yields two

ATP molecules.

34. Which statement about starch metabolism is FALSE?

a. Starch is synthesized from an intermediate product of the Calvin cycle.

b. Oxidative respiration of the breakdown products of starch occurs in the cytosol

(the soluble aqueous portion of the cytoplasm).

c. Hydrolytic enzymes catalyse the breakdown of starch to free sugars.

d. Starch is a linear and/or branched polymer of glucose.

e. Amylase enzyme from human saliva degrades starch.

35. What happens each day when the leaf of a plant is first exposed to light?

a. The epidermal cells on the upper surface of the leaf begin to photosynthesize.

b. Carbon dioxide diffuses out of the leaf.

c. The cells of the spongy mesophyll begin to take up carbon dioxide, and are the

only cells that are able to do so.

Page 66: 2001 -1995

d. The concentration of oxygen in the intercellular spaces of the leaf decreases.

e. The guard cells become more turgid and the stomatal apertures open.

36. Which statement about the light reactions of photosynthesis is FALSE?

a. Photosystems I and II are located in the stroma of the chloroplast.

b. Photosystems I and II are linked by a chain of electron carriers that is similar

to that found in mitochondria.

c. During photophosphorylation, water is oixidized to form H+ and O2, yielding

electrons to Photosystem II.

d. Chlorophyll pigments have an absorption spectrum with pronounced peaks in

the red and blue wavelengths.

e. Protons diffuse through protein channels which are ATP-synthase molecules.

37. The graph below shows the relationship between photosynthetic rate and temperature. Based

on these results, which species is best adapted to arctic conditions where the mean temperature

does not exceed 8oC during the growing season?

a. Species A

b. Species B

c. Species C

d. Species D

e. All species are equally well adapted.

38. In the veins of a spinach leaf:

a. the water in the xylem vessels moves when the water potential of the

atmosphere is less negative than the water potential of the vessels themselves.

Page 67: 2001 -1995

b. the water in the xylem vessels is under a positive hydrostatic pressure.

c. the water in the xylem vessels is attracted to nonpolar molecules within the cell

walls.

d. the solution in the phloem sieve tubes is under a negative hydrostatic pressure

(tension).

e. the water potential of the phloem sieve tube elements becomes more negative

as carbohydrates are "loaded" into the elements by active transport.

39. The susceptibility to many cancers is inherited, despite the fact that those who contract the

disease usually die quickly. The evidence shows that the usual explanation for this is that people

who are at a genetic risk for cancer inherit:

a. a deficient RNA editing mechanism.

b. a predisposition (tendency) to avoid foods that prevent cancer.

c. one of two or more mutations necessary to cause the disease.

d. a predisposition to expose themselves to chemical carcinogens, such as those

found in tobacco.

e. a single cancer cell, which may or may not divide.

40. When a DNA molecule is replicated before mitosis the result is:

a. two DNA molecules, one of which carries all of the original DNA molecule,

while the other is newly synthesized.

b. two DNA molecules, each of which contains half of the original DNA

molecule.

c. two DNA molecules, one of which carries all of the original DNA molecule,

while the other is newly synthesized, plus the RNA template used as an

intermediate.

d. two DNA molecules, each of which contains half of the original DNA

molecule, plus the RNA template used as an intermediate.

e. two chromosomes.

41. From a scientific point of view, which of the following properties would help make a gene a

practical target for gene therapy in a human?

i. The product of the gene is disseminated outside the tissue of origin.

ii. The gene is transcribed primarily in rapidly dividing cells.

iii. The gene is the cause of a very common genetic disease.

iv. The defect to be corrected is the result of a simple base pair difference.

Page 68: 2001 -1995

a. i and ii b. i, ii, and iv c. i and iiid. ii and iiie. iii and iv

42. Which of the following changes to a DNA molecule is least likely to result in a deleterious

mutation?

a. Insertion of a transposable element in a coding region.

b. Deletion of a base pair in a coding region.

c. Change of a base pair in the first codon of a coding region.

d. Change of the first base pair of a codon.

e. Change of the third base pair of a codon.

43. The goal of the polymerase chain reaction is to:

a. speed up protein synthesis for the production of new drugs.

b. create many copies of a DNA sequence which is initially very rare.

c. create many copies of messenger RNA molecules.

d. investigate the properties of organisms which normally grow at very high

temperatures.

e. create DNA probes.

44. Efforts to repeat the cloning of Dolly the sheep have been unsuccessful. The scientists who

claim to have cloned Dolly have been challenged to produce additional evidence that Dolly is

really derived from an adult cell (as originally claimed), rather than a fetal cell which might have

contaminated the experiment (as the sceptics suggest). Which of the following could provide

evidence that Dolly was created from an adult cell, rather than a fetal cell?

a. Dolly's DNA fingerprints.

b. The heterozygosity of random pieces of Dolly's DNA.

c. The melting point of Dolly's DNA.

d. The length of Dolly's telomeres.

e. Dolly's biological clock.

45. Although the seal and the penguin both have streamlined, fish-like bodies with a layer of

insulating fat, they are not closely related. This similarity results from:

a. homologous evolution.

Page 69: 2001 -1995

b. convergent evolution.

c. adaptive radiation.

d. coevolution.

e. parallel evolution.

46. What does the term "reproductive isolation" refer to?

a. An individual is unable to fertilize itself.

b. Genes are not exchanged between two populations.

c. Individuals from two populations never mate.

d. Individuals from two populations never produce offspring.

e. Individuals are solitary breeders.

47. Why is it unlikely that humans will undergo speciation in the future?

a. Migration among populations is high.

b. Natural selection is no longer operating on humans.

c. Random effects have become more important in large populations.

d. The environment on Earth is being controlled and stabilized.

e. The human species has exhausted nearly all of its genetic potential.

48. For a trait that is controlled by two alleles at a single locus, the frequency of the dominant

allele is 0.6. What is the genotype frequency of heterozygous individuals, assuming the population

is at Hardy-Weinberg equilibrium?

a. 0.16

b. 0.24

c. 0.36

d. 0.48

e. 0.72

49. Which prediction from the fossil record is in agreement with the theory of evolution?

a. Jawed fishes are found deeper in rock strata (layers) than jawless fishes.

b. Land animals appear earlier than land plants in the fossil record.

c. Unicellular organisms are first found in strata above multicellular fossils.

d. Prokaryote fossils appear earlier than eukaryote fossils.

e. Reptiles appear earlier than insects in the fossil record.

Page 70: 2001 -1995

50. Which of the following is NOT a correct match of hormone and function?

a. Luteinizing hormone - production of testosterone by the testes.

b. Prolactin - milk production in the mammary glands.

c. Glucagon - synthesis of glycogen in liver cells.

d. Oxytocin - contraction of uterine muscles.

e. Adrenalin (epinephrine) - inhibition of insulin release by the pancreas.

51. Which statement about the vertebrate nervous system is FALSE?

a. Relaxation of a muscle is caused by nerve impulses in inhibitory neurons

going to the muscle from the spinal cord.

b. After completely severing the brain from the spinal cord, reflex withdrawal of

the foot from a painful stimulus could still occur.

c. In a simple reflex, the sequence followed by information is: sensory receptor,

sensory neuron, interneuron, motor neuron, muscle cells.

d. The sympathetic nervous system activates the body's responses to stress, for

example, by increasing heart beat and decreasing blood flow to the gut.

e. The spinal cord and the brain contain many synapses and both function in the

processing of information.

52. If you were to insert a tiny heating probe into the thermoregulatory centre of the hypothalamus

and use it to raise the temperature of the hypothalamus slightly, what is most likely to occur?

a. Shivering

b. Decreased circulation in the skin

c. Decreased activity of the sweat glands

d. A drop in body temperature

e. Increased muscular activity

53. Suppose you are developing a new drug, and have found that when it is administered in

humans there is a substantial increase in the volume of urine produced. When you administer

antidiuretic hormone (ADH, or vasopressin) at the same time, the volume of urine returns to

normal. Which hypothesis best fits these observations? The new drug:

a. blocks the receptors for ADH on the collecting ducts of the kidney.

b. blocks the release of ADH from the pituitary.

c. mimics the action of ADH.

d. damages the kidney.

e. decreases blood pressure.

Page 71: 2001 -1995

54. Along with nicotine, cigarette smokers receive tars, phenols, hydrocarbons, arsenic, and many

other chemicals. All of the following are effects of inhaling tobacco EXCEPT:

a. narrowing or hardening of blood vessels in the heart and brain.

b. stomach ulcers, due to an increased acidity in the stomach.

c. a higher frequency of respiratory infections (e.g., colds, pneumonia).

d. a higher risk of cancer, including cancer of the lungs, mouth, larynx, bladder,

and kidneys.

e. stimulation of the central nervous system, including increased alertness.

55. Where would one expect to find the highest concentration of PCBs (polychlorinated biphenyls,

an organic contaminant) in a lake?

a. Water

b. Small fish

c. Sediment

d. Phytoplankton

e. Zooplankton

56. You were asked to measure the density of rats on an island using the mark-recapture technique.

You captured 200 rats, marked them with tags and released them. A week later you sampled 500

rats and measured how many of these rats were marked. This ratio allows you to estimate the size

of the rat population on the island, as shown in the equation below. Which of the following must

occur in order for your estimate of population size to be accurate?

Number marked and released     =     Number marked in recaptured sample

     Total population size                       Total number in recaptured sample

a. The marked rats must not move far from the region where you released them.

b. The marked rats must represent a small proportion of the total population.

c. The marked rats must mix freely with the rest of the population.

d. The death rate in the rat population must be low.

e. The rats on the island must be territorial.

57. Carbon dioxide is called a "greenhouse" gas because:

a. its concentration is always high in greenhouses.

b. it is involved in photosynthesis.

Page 72: 2001 -1995

c. it absorbs infra-red radiation.

d. it emits visible radiation (i.e., light).

e. it absorbs and emits ultra-violet radiation.

58. As the number of individuals approaches the carrying capacity of a population, which of the

following is predicted by the sigmoidal growth curve?

a. Population density will increase exponentially.

b. Population density will decrease exponentially.

c. Population growth rate will increase.

d. Population growth rate will decrease.

e. Population biomass will remain the same.

59. In most ecosystems, the biomass of a trophic level is higher than the biomass of its predators,

as illustrated below by an upright pyramid of biomass. In the open ocean, however, the biomass of

primary producers (microscopic algae) is often lower than the biomass of higher trophic levels (for

example, zooplankton and fish), as illustrated below by an inverted pyramid of biomass. In the

open ocean, how can there be enough food to support the higher trophic levels?

Note: The width of the bars indicates the amount of biomass at each trophic level.

a. The microscopic primary producers are a source of food of high quality.

b. The microscopic primary producers have high rates of growth and

reproduction.

c. The microscopic primary producers are very abundant.

d. The higher trophic levels are cold-blooded animals which do not require much

food.

e. The higher trophic levels are efficient predators.

60. One of the consequences of El Niño is a decline in the number of fish caught along the coasts

of Ecuador and Peru. This decline is primarily because:

Page 73: 2001 -1995

a. of poor weather conditions.

b. of increased salinity of surface waters along the coast.

c. warmer water attracts more predators (for example, dolphins and seals).

d. warmer water increases fish mortality.

e. cold nutrient-rich water does not upwell to the surface along the coast.

The University of Toronto National Biology Competition

1998 Examination

Answer Key

1. b 2. a 3. c 4. c 5. e

6. a 7. d 8. b 9. c 10. e

11. c 12. d 13. e 14. e 15. c

16. d 17. a 18. e 19. a 20. b

21. a 22. d 23. b 24. b 25. e

26. a 27. a 28. c 29. e 30. c

31. b 32. d 33. a 34. b 35. e

36. a 37. d 38. e 39. c 40. b

41. a 42. e 43. b 44. d 45. b

46. b 47. a 48. d 49. d 50. c

51. a 52. d 53. b 54. e 55. c

56. c 57. c 58. d 59. b 60. e

Note about questions 33 and 36

The University of Toronto Ontario Biology Competition

Page 74: 2001 -1995

1997 Examination Time: 90 minutes

Number of Questions: 60

NOTE:   PLEASE READ

If you click on the answer links and the answer is not displayed, do the following. From the toolbar, click on Options then Network Preferences. Click on the Languages tab and ensure that the "JavaScript Enabled" box is selected.

When you click on the "The Answer is..." link, a window will pop up.  You MUST close it, by clicking on "Back to Test".   If you do not do this, subsequent windows displaying the answer will not be visible.

This document is best viewed with Netscape 3.0.

Answer Key

General Instructions

Do not open this booklet until you are instructed to do so.

Print your name at the top of this booklet.

Indicate all of your answers to the questions on the separate Response Form. No credit

will be given for anything written in this booklet, but you may use the booklet for notes or

rough work. No additional time will be given after the exam to transfer your answers to

the Response Form.

After you have decided which of the suggested answers is best, COMPLETELY fill in the

corresponding circle on the Response Form. Give only one answer to each question. If

you change an answer, be sure that the previous mark is erased completely.

Use your time effectively. Do not spend too much time on questions that are too difficult.

Go on to other questions and come back to the difficult ones later if you have time. It is

not expected that everyone will be able to answer all questions.

Good luck and have fun!

Should you guess the answers to questions about

which you are not certain?

Page 75: 2001 -1995

Since your score on the exam is based on the number of questions you answered correctly minus one-third of the number you answered incorrectly, it is improbable that guessing will improve your score (it is more likely to lower your score). (No points are deducted or awarded for unanswered questions).  However, if you are not sure of the correct answer but have some knowledge of the question and are able to eliminate one or more of the answer choices, then your chance of getting the right answer is improved, and it may be advantageous to answer such a question.

1. Which statement is FALSE?

a. A ribosome is smaller than a centriole.

b. The nucleus has one membrane surrounding it.

c. Plant cells contain mitochondria.

d. A centriole contains microtubules.

e. The largest organelle in a typical plant cell is the vacuole.

[The Answer is...]

2. A scientist put some plant cells in a solution of glucose and, after 30 minutes, measured the

concentration of glucose inside the cells. When she repeated the experiment using double the

concentration of glucose in the solution, the concentration of glucose inside the cells also doubled.

Which of the following principles does this experiment demonstrate?

a. Glucose enters plant cells by active transport.

b. Glucose enters plant cells by endocytosis.

c. Glucose enters plant cells by some type of diffusion.

d. Plant cells cannot synthesize glucose.

e. Glucose moves into plant cells by osmosis.

[The Answer is...]

3. Which statement best explains why protozoans that live in pond water must use exocytosis to

get rid of excess water in their cytoplasm?

a. The pond water is hypotonic to the protozoan cytoplasm.

b. The pond water is hypertonic to the protozoan cytoplasm.

c. The pond water is isotonic to the protozoan cytoplasm.

d. Water cannot diffuse across the protozoan plasma membrane.

e. Protozoans take up water by endocytosis; therefore, they must expel it by

Page 76: 2001 -1995

exocytosis.

[The Answer is...]

4. Plant cells can often grow larger than animal cells because:

a. photosynthesis produces more energy than does respiration.

b. plant cells need less food than animal cells.

c. plants use their energy for growth while animals spend much of their energy

on movement.

d. plant cells have a cell wall which makes their surface to volume ratio very

large.

e. the large central vacuole of plant cells reduces the depth of the cytoplasm and

allows entering molecules to rapidly diffuse from the plasma membrane to all

parts of the cytoplasm.

[The Answer is...]

5. Which of the following cell types provide mechanical support for plants?

a. Tracheids, sclerenchyma fibres, collenchyma

b. Sclerenchyma fibres, cork, cortex

c. Sieve tubes, vessel elements, epidermis

d. Sclerenchyma fibres, pith cells, epithelial cells

e. Pith, cortex, xylem

[The Answer is...]

6. Water can move from the soil and up through the world's tallest living tree (which is 110 metres

high) by which of the following process(es)?

a. Osmosis

b. Root pressure

c. Pressure flow

d. Adhesion and cohesion of water molecules

e. All of the above

[The Answer is...]

7. Which statement about the function of the Casparian strip is CORRECT?

Page 77: 2001 -1995

a. It prevents excess transpiration from leaves.

b. It regulates ion movement into the root vascular cylinder.

c. It prevents disease organisms from invading the plant.

d. It prevents ions from leaking out of the xylem into leaves.

e. It is the pathway for nutrient transfer from xylem to phloem.

[The Answer is...]

8. What path does a carbon atom take in going from the atmosphere into the cell wall of a root cap

cell?

a. Stoma---proplastid---chloroplast---vessel element---procambium---apical

meristem---root cap

b. Stoma---chloroplast---phloem---apical meristem---procambium---root cap

c. Stoma---chloroplast---vessel element---procambium---apical meristem---root

cap

d. Stoma---chloroplast---sieve tube---procambium---apical meristem---root cap

e. All of these paths are possible.

[The Answer is...]

9. The plant growth regulator involved in aging and ripening of fruit is:

a. cytokinin.

b. auxin.

c. gibberellin.

d. abscisic acid.

e. ethylene.

[The Answer is...]

10. Which of the following provides the best evidence that all known forms of life descended from

a common ancestor?

a. DNA

b. Chloroplasts

c. The genetic code

d. The homeobox sequence

e. Mitochondria

[The Answer is...]

Page 78: 2001 -1995

11. Two cells which are exactly like the original cell result from:

a. gamete formation.

b. meiosis.

c. mitosis.

d. sexual reproduction.

e. syngamy.

[The Answer is...]

12. Imagine that in a plant, hairy leaves are dominant to smooth leaves, and blue flowers are

dominant to white. You cross a pure breeding hairy-leaved, blue-flowered strain with a pure

breeding smooth-leaved, white-flowered strain. You allow the offspring to self-fertilize, and

observe that the F2 offspring are 69% hairy-leaved and blue-flowered, 19% smooth-leaved and

white-flowered, 6% hairy-leaved and white-flowered, and 6% smooth-leaved and blue-flowered.

Based on these observations you can conclude that the two genes:

a. are in the same biochemical pathway.

b. segregate independently.

c. exhibit typical Mendelian ratios.

d. are on the same chromosome.

e. show epistatic interactions.

[The Answer is...]

13. Recently, scientists induced a single diploid cell from an adult sheep (the "mother") to grow

into a lamb, named Dolly. How closely are Dolly and her "mother" related?

a. They are completely unrelated.

b. They are about as related as a brother and a sister would be.

c. They are about as related as fraternal twins.

d. They are about as related as identical twins.

e. They are completely identical genetically.

[The Answer is...]

14. You are moving to Africa and you have heard that the Green Fever virus disease is common

there. Before you leave, you visit your doctor for advice. Which of the following is your doctor

most likely to recommend? You should:

a. take a large supply of antibiotics with you.

Page 79: 2001 -1995

b. wash your food in an antiseptic before you eat it.

c. get vaccinated against the disease.

d. not worry as viruses only cause diseases in plants.

e. get regular blood tests which will use the light microscope to look for virus

particles.

[The Answer is...]

15. A hospital technician, while doing some routine culturing of micro-organisms in a lab, noticed

a bacterial colony growing on a culture medium containing three different antibiotics. He

identified the bacterium as one that did not cause a human disease, but he still reported his

observation to the hospital administration. Which statement is CORRECT? He was worried

because:

a. he had no way of killing this bacterium now that it was resistant to antibiotics.

b. resistance to antibiotics could be transferred to disease-causing bacteria by

transduction or conjugation.

c. the bacterium might feed on the antibiotics and therefore be able to grow in

people taking these antibiotics.

d. there should be no bacteria inside a hospital.

e. if people accidentally ingested the bacteria with their food, they would become

resistant to the antibiotic.

[The Answer is...]

16. Which statement is FALSE?

a. Viruses primarily contain nucleic acids and protein.

b. All viruses are parasites.

c. Bacteria are usually less than 2 micrometres in diameter.

d. Autotrophic bacteria can use carbon dioxide as their sole source of carbon.

e. Bacteria can only fix nitrogen when they are inside the root nodules of

leguminous plants, such as peas and beans.

[The Answer is...]

17. Which statement is FALSE?

a. Pepsin is an enzyme that acts on peptide chains to digest proteins.

b. The enzyme enterokinase produces the protein-digesting enzyme trypsin from

trypsinogen.

Page 80: 2001 -1995

c. The bile helps digestion by breaking fat into tiny droplets.

d. The pancreas produces more than 20 different digestive enzymes.

e. Salivary amylase starts the process of protein digestion in the mouth.

[The Answer is...]

18. Which of the following best represents the path taken by an amino acid molecule after it is

ingested?

a. mouth---salivary glands---oesophagus---small intestine---pancreas

b. mouth---oesophagus---stomach---duodenum---liver

c. mouth---duodenum---stomach---small intestine---colon

d. mouth---oesophagus---pancreas---jejunum---stomach

e. mouth---jejunum---liver---colon---small intestine

[The Answer is...]

19. Which of the following statements about the gills of fish is FALSE?

a. The flow of water over the gill is in the same direction as the flow of blood in

the filament.

b. Oxygen gas dissolved in the water diffuses into blood vessels of the gill.

c. Blood flows into and out of each filament of the gill in separate vessels.

d. Decreasing the rate of movement of water over the gills would decrease

oxygen uptake.

e. Most fish actively force water over their gills by movement of the operculum.

[The Answer is...]

20. In a mammalian lung the rate at which oxygen could be obtained from the air would increase

if:

a. tidal volume decreased.

b. the cells lining the alveoli and capillaries were thinner.

c. blood haemoglobin content were lower.

d. you ascended to a higher altitude.

e. the alveoli were larger.

[The Answer is...]

21. Which statement is FALSE?

Page 81: 2001 -1995

a. Capillaries in muscle are usually empty of blood.

b. The mammalian heart will continue to beat when its nerve supply is cut.

c. In vertebrates, veins can be distinguished from arteries because veins carry

deoxygenated blood.

d. Water and small molecules can easily pass through the walls of most

capillaries.

e. Circulation of blood in the coronary arteries is greatest during diastole, when

the heart is filling.

[The Answer is...]

22. If a molecule of carbon dioxide released into the blood in the foot of a human foetus is exhaled

through the mouth of the mother, it will NOT travel through the:

a. right atrium of the foetus.

b. right atrium of the mother.

c. left ventricle of the foetus.

d. left ventricle of the mother.

e. placenta.

[The Answer is...]

23. Which statement is FALSE?

a. Cell division to form a blastocyst begins after the fertilized egg is implanted in

the endometrium of the uterus.

b. The placenta contains cells derived both from the embryo and the mother.

c. Follicle stimulating hormone (FSH) stimulates the production of sperm in the

male testes.

d. Human males can continue to produce sperm throughout life, whereas females

stop producing eggs during menopause.

e. Some female mammals only release eggs from their ovaries when copulation

occurs.

[The Answer is...]

24. Trace a sperm cell from the structure where it is produced to fertilization of the egg:

seminiferous tubules (1); vas deferens (2); uterus (3); fallopian tube (4); vagina

(5); epididymis (6); and urethra (7).

Page 82: 2001 -1995

a. 6, 1, 2, 7, 5, 3, 4b. 1, 6, 2, 7, 5, 3, 4 c. 1, 6, 2, 7, 5, 4, 3d. 1, 2, 6, 7, 5, 3, 4e. 1, 2, 6, 7, 5, 4, 3

[The Answer is...]

25. The atomic number of an element is the same as the number of:

a. neutrons in each atom.

b. neutrons + protons in each atom.

c. protons + electrons in each atom.

d. protons in each atom.

e. electrons in an ion.

[The Answer is...]

26. All of the following molecules are carbohydrates EXCEPT:

a. lactose.

b. haemoglobin.

c. cellulose.

d. glycogen.

e. starch.

[The Answer is...]

27. What is the difference between covalent and ionic bonds?

a. Covalent bonds are the sharing of neutrons; ionic bonds are the sharing of

electrons.

b. Covalent bonds are the electric attraction between two atoms; ionic bonds are

the sharing of electrons between atoms.

c. Covalent bonds are the sharing of protons between atoms; ionic bonds are the

electric attraction between two atoms.

d. Covalent bonds are the sharing of protons between atoms; ionic bonds are the

sharing of electrons between two atoms.

e. Covalent bonds are the sharing of electrons between atoms; ionic bonds are the

electric attraction between two atoms.

[The Answer is...]

Page 83: 2001 -1995

28. What would be an expected consequence of changing one amino acid in a particular protein?

a. The primary structure would be changed.

b. The tertiary structure would be changed.

c. The biological activity of this protein might be altered.

d. The number of amino acids present would stay the same.

e. All of the above are expected.

[The Answer is...]

29. The essential characteristic of a polar molecule is that it:

a. contains oxygen.

b. contains ions as part of the structure.

c. has an asymmetrical distribution of electrical charge.

d. is formed at extremely low temperatures.

e. contains double or triple bonds.

[The Answer is...]

30. A scientist produced some mutant fungal cells that, under normal growth conditions, were

much smaller, and which grew much more slowly, than normal cells. When the mutant and normal

cells were each grown in a glucose solution in a tightly sealed container and examined after 1

week, the two types of cells looked identical and had grown to the same extent. Which statement

best explains these results?

a. Each mutant fungal cell was lacking a Golgi apparatus.

b. Glucose is poisonous to fungal cells.

c. Glucose could not be used as an energy source by mutant fungal cells.

d. The mutant fungal cells had abnormal mitochondria.

e. The mutant fungal cells had abnormal lysosomes.

[The Answer is...]

31. Animal cells were placed in a water solution containing sodium and potassium ions. After 10

minutes, the concentration of sodium had increased in the solution, but the concentration of

potassium had decreased. When the experiment was repeated with glucose added to the solution,

the concentration of sodium in the solution increased faster, but the decrease in potassium was the

same as in the first experiment. Which statement best explains these results?

a. Sodium leaves the cells, and potassium enters the cells, by passive transport.

Page 84: 2001 -1995

b. Sodium is excreted from the cells by active transport and potassium enters the

cells by passive transport.

c. Glucose enters the cells by passive transport.

d. Sodium leaves the cells by passive transport and potassium enters the cells by

active transport.

e. The plasma membrane is more permeable to sodium ions than potassium ions.

[The Answer is...]

32. Which statement is FALSE?

a. Glucose-6-phosphate is an intermediate in glycolysis.

b. Pyruvate from glycolysis enters the mitochondrion and is converted to acetyl

coenzyme A.

c. Some energy from ATP is required for glucose to enter the glycolysis process.

d. For each glucose molecule broken down during aerobic respiration, the citric

acid cycle produces more ATP molecules than does the electron transport chain.

e. The energy to make ATP comes from a proton gradient across the inner

mitochondrial membrane, which is made by passing electrons through the

electron transport chain.

[The Answer is...]

33. Fats are a better source of energy than carbohydrates because:

a. fats are digested better than carbohydrates.

b. the oxidation of fats produces larger quantities of ATP.

c. fats are converted to fatty acids that directly enter the citric acid cycle.

d. fats enter cells faster than carbohydrates.

e. fats can be broken down by glycolysis without entering the mitochondrion.

[The Answer is...]

34. Hibernating animals (that sleep all winter) have tissues containing mitochondria with a

membrane protein that accelerates electron transport while blocking the synthesis of ATP. What is

the consequence of this?

a. Energy is saved because glycolysis and the citric acid cycle shut down.

b. Hibernating animals do not have enough energy to keep warm in cold weather,

so they have to sleep through the entire winter.

c. Pyruvate is converted to lactic acid by anaerobic fermentation.

Page 85: 2001 -1995

d. Hibernating animals can synthesize fat instead of wasting energy on

respiration.

e. The energy of respiration is converted to heat.

[The Answer is...]

35. An investigator wanted to test the mass-flow theory of phloem transport. She knew that a

growing melon must receive a large flow of organic material by way of the slender vine.

Therefore, she thought if she cut the vine, a good deal of phloem sap should leak out of the stump.

But when she tried, only a small droplet of phloem sap emerged from the stump. What is most

likely to have happened?

a. The mass-flow theory was wrong.

b. The flow stopped because the melon no longer provided the needed pull.

c. The flow stopped because without the melon, phloem transport was no longer

needed.

d. The sieve pores probably got plugged with P-protein.

e. The callose system stopped producing ATP needed for transport.

[The Answer is...]

36. Which statement comparing the biochemical processes of photosynthesis and cellular

respiration is FALSE?

a. Both biochemical processes take place in specialized organelles that have

complex systems of internal membranes.

b. ATP synthesis in both processes relies on the chemiosmotic mechanism,

involving the pumping of protons through a membrane.

c. Both processes involve the passing of electrons from carrier to carrier in a

series of oxidation-reduction reactions which liberate energy.

d. The initial source of electrons which pass from carrier to carrier are from high-

energy food molecules in both processes.

e. Oxygen is an end-product of photosynthesis and carbon dioxide is an end-

product of cellular respiration.

[The Answer is...]

37. Which statement about the Calvin cycle is FALSE?

a. It is used by C4 plants to concentrate carbon dioxide.

b. The initial carboxylation reaction is catalysed by the enzyme RUBP

Page 86: 2001 -1995

carboxylase (Rubisco).

c. The reduction of 3PG to a sugar phosphate requires ATP, NADPH+, and H+.

d. It is a process which involves enzymes that are light activated.

e. The Calvin cycle operates in C3, C4, and Crassulacean Acid Metabolism

(CAM) plants.

[The Answer is...]

38. Which statement concerning stomatal function is FALSE?

a. Stomata are open when the turgor pressure of the guard cells decreases.

b. When stomata are about to open, potassium ions are actively transported into

the guard cells from the surrounding cells of the epidermis.

c. To close stoma, potassium ions diffuse passively out of the guard cells.

d. Low levels of carbon dioxide stimulate the opening of stomata.

e. High levels of water stress promote stomatal closure.

[The Answer is...]

39. Which statement best explains why C4 grasses often do better than C3 grasses in hot, dry

environments?

a. C4 grasses open their stomates at night.

b. C4 grasses have nearly eliminated photorespiration.

c. C4 grasses generate a positive turgor pressure under high temperatures.

d. The enzymes for photorespiration in C4 grasses are inactivated at high

temperatures.

e. The rate of cellular respiration is higher for a C3 grass than for a C4 grass at

higher temperatures.

[The Answer is...]

40. In a charged transfer RNA, the nucleotide bound to the amino acid is adenosine (A), and the

next two nucleotides are cytosines (C). What can you tell about the DNA codon to which this

transfer RNA corresponds?

a. The codon is TGG.

b. The codon is ACC.

c. The codon is UGG.

d. The first position is A, but you can't tell about the others from the information

given.

Page 87: 2001 -1995

e. You can't tell anything about the codon from the information given.

[The Answer is...]

41. Approximately what percentage of the DNA in the human genome is both transcribed and

translated?

a. 100%

b. 70%

c. 50%

d. 20%

e. 3%

[The Answer is...]

42. Which of the following is true of all cancer cells? The rapid growth and division of cancer

cells is caused by:

a. bacterial infection.

b. breakdown of normal gene regulation.

c. viruses.

d. changes in the intracellular hormone receptors.

e. toxic chemicals.

[The Answer is...]

43. Which application of genetics has had the greatest impact on the well being of humans to date?

a. Plant breeding

b. Animal breeding

c. Genetic engineering

d. In vitro fertilization

e. Genetic counselling

[The Answer is...]

44. If you were on a jury trying to determine whether a man was the father of a child, which kind

of evidence should you consider to be most informative?

a. Blood type

b. Skin colour

Page 88: 2001 -1995

c. DNA fingerprint

d. Enzyme electrophoretic type

e. Fingerprints

[The Answer is...]

45. New strains of bacteria are becoming resistant to antibiotics at a high rate. Which genetic

process is now responsible for most of this increase in the proportion of resistant bacteria?

a. Transformation

b. Transduction

c. Mutation

d. Insertion

e. Transposition

[The Answer is...]

46. Which statement best defines evolution?

a. The close resemblance between parents and their offspring

b. Difference between individuals in survival

c. Individuals in two populations look different

d. Change in the phenotype of an individual through time

e. Change in genetic composition of a population

[The Answer is...]

47. What is the main target of natural selection?

a. The population

b. Individual phenotype

c. Individual genotype

d. Individual gene

e. The species

[The Answer is...]

48. Scientists consider the horse and the donkey to be different species, but when individuals of

these two species mate they produce a strong offspring called a mule. According to the biological

species concept, should scientists reclassify the horse and donkey as belonging to the same

species?

Page 89: 2001 -1995

a. Yes, horses and donkeys must be considered one species if they can mate and

produce an offspring.

b. Yes, because horses and donkeys are morphologically similar.

c. No, because the mule cannot breed with either horse or donkey.

d. No, because this cross-mating is rare in nature.

e. No, horses and donkeys are probably different genetically, which is the only

factor used in determining species.

[The Answer is...]

49. A biologist sequenced the cytochrome c enzyme in a range of animals and compared the amino

acid (AA) differences between pairs of species. Which of the following results would be most

consistent with modern systematics?

a. Chimpanzee/rhesus monkey 23 AA differences; horse/chimpanzee 2 AA

differences; chimpanzee/shark 8 AA differences.

b. Chimpanzee/rhesus monkey 23 AA differences; horse/chimpanzee 23 AA

differences; chimpanzee/shark 23 AA differences.

c. Chimpanzee/rhesus monkey 12 AA differences; horse/chimpanzee 8 AA

differences; chimpanzee/shark 3 AA differences.

d. Chimpanzee/rhesus monkey 2 AA differences; horse/chimpanzee 12 AA

differences; chimpanzee/shark 23 AA differences.

e. Chimpanzee/rhesus monkey 2 AA differences; horse/chimpanzee 23 AA

differences; chimpanzee/shark 12 AA differences.

[The Answer is...]

50. If one gamete in five carries a recessive allele, what must be the frequency of the homozygous

recessive genotype in a population at Hardy-Weinberg equilibrium?

a. 4%

b. 16%

c. 20%

d. 60%

e. 80%

[The Answer is...]

51. Which statement is FALSE?

a. Even if the sodium-potassium pump of a nerve cell brings about ion exchange,

Page 90: 2001 -1995

the fact that the membrane is naturally more permeable to sodium means that

more sodium will leak out than potassium in.

b. The energy for the sodium-potassium pump probably comes from the

breakdown of ATP to ADP.

c. Axons usually synapse with the dendrites or the cell body of the next neuron in

a sequence.

d. In vertebrates, myelinated nerve fibers generally transmit impulses more

rapidly than do unmyelinated fibers of comparable diameter.

e. Neurons do not make direct connections with one another; they are separated

by a space known as the synaptic cleft.

[The Answer is...]

52. Which of the following would NOT result from the release of adrenalin (epinephrine)?

a. Decreased blood flow to skin

b. Increased oxygen consumption

c. Rise in blood pressure

d. Increased conversion of glycogen to glucose

e. Increased blood flow to intestine

[The Answer is...]

53. Suppose you were a neuroscientist and had been given a sample of a new snake venom. You

test its effect on action at a synapse, and find that it increases the magnitude of the normal

depolarizing excitatory response. The most likely explanation for this is that the venom is:

a. blocking release of the neurotransmitter from the vesicles.

b. binding with the neurotransmitter receptors to interfere with neurotransmitter

binding.

c. binding with the neurotransmitter receptors to mimic the action of the

neurotransmitter.

d. acting to break down the neurotransmitter in the synaptic cleft.

e. preventing the presynaptic action potential from arriving at the synapse.

[The Answer is...]

54. Which statement is FALSE?

a. Steroid hormones act by passing through the cell membrane and affecting gene

transcription.

Page 91: 2001 -1995

b. Adrenaline (epinephrine) acts on cell membrane receptors to stimulate

production of second messengers.

c. The anterior pituitary releases several hormones which act on other endocrine

glands.

d. Insulin is produced in the islets of Langerhans in the spleen, when blood sugar

levels fall.

e. The posterior pituitary releases a hormone that regulates kidney activity and

blood pressure.

[The Answer is...]

55. Which statement is FALSE?

a. The kidney produces a concentrated urine by establishing a high concentration

of salt and urea surrounding the collecting ducts.

b. The hypothalamus is a region of the hind brain important in regulating

cardiovascular function, such as heart rate and blood pressure.

c. In typical reflex arcs, impulses in sensory neurons activate motor neurons via

interneurons.

d. A drop in body temperature is countered by measures such as increasing

metabolic rate and shutting down circulation in the skin. This is an example of

negative feedback.

e. Both the brain and spinal cord are connected to nerves that carry motor and

sensory information to and from the viscera.

[The Answer is...]

56. Approximately 1.5 million species have been identified and described by scientists. Which

group contains the most species worldwide?

a. Bacteria

b. Fungi

c. Vascular plants

d. Insects

e. Mammals

[The Answer is...]

57. Which nutrient is most likely to limit primary production in terrestrial ecosystems in Canada?

a. Carbon

Page 92: 2001 -1995

b. Sulfur

c. Phosphorus

d. Nitrogen

e. Potassium

[The Answer is...]

58. Two insect species were used in a laboratory experiment. For one treatment, both species were

grown by themselves (in separate chambers) on a suitable food source. For the second treatment,

the two species were grown together (in the same chamber) on the same type and amount of food

as in the first treatment. The figure below shows the results (the number of individuals of each

species in the two treatments) at the end of the experiment. Based on these results the two species

should be classified as:

a. competitors.

b. antagonists.

c. mutualists.

d. predators or pathogens.

e. commensalists.

[The Answer is...]

59. In a hypothetical developing nation, new government policies and international aid have

resulted in improved family planning and decreased death rates. As a result, the average number of

Page 93: 2001 -1995

children per female, the average age of reproduction, and the average life expectancy are now the

same as in Canada. If we ignore changes in population size because of immigration and

emigration, the population growth rate of the developing nation will now:

a. be the same as Canada's.

b. decrease, attaining Canada's growth rate within 10 years.

c. remain higher than Canada's because of climate.

d. remain higher than Canada's because of economic differences between the

developing nation and Canada.

e. remain higher than Canada's until the population has reached a stable

distribution of age classes.

[The Answer is...]

60. If we wish to manage a deer population so that a very high number of deer can be harvested,

we should manage the population so that:

a. the number of deer is far enough below the carrying capacity to support high

birth and growth rates.

b. the number of deer is at the carrying capacity to provide the largest sustainable

deer population.

c. the deer are rare and have little contact with each other.

d. the number of deer slightly exceeds the carrying capacity so that the excess

can be harvested.

e. the number of deer greatly exceeds the carrying capacity to provide a large

number of excess individuals.

The University of Toronto Ontario Biology Competition

1997 Examination

Answer Key

1. b 2. c 3. a 4. e 5. a

6. d 7. b 8. d 9. e 10. c

11. c 12. d 13. d 14. c 15. b

16. e 17. e 18. b 19. a 20. b

Page 94: 2001 -1995

21. c 22. d 23. a 24. b 25. d

26. b 27. e 28. e 29. c 30. d

31. b 32. d 33. b 34. e 35. d

36. d 37. a 38. a 39. b 40. e

41. e 42. b 43. a 44. c 45. a

46. e 47. b 48. c 49. d 50. a

51. a 52. e 53. c 54. d 55. b

56. d 57. d 58. c 59. e 60. a

The University of Toronto Ontario Biology Competition

1996 Examination Time: 90 minutes

Number of Questions: 60

Answer Key

General Instructions

Do not open this booklet until you are instructed to do so.

Print your name at the top of this booklet.

Indicate all of your answers to the questions on the separate Response Form. No credit

will be given for anything written in this booklet, but you may use the booklet for notes or

rough work. No additional time will be given after the exam to transfer your answers to

the Response Form.

After you have decided which of the suggested answers is best, COMPLETELY fill in the

corresponding circle on the Response Form. Give only one answer to each question. If

you change an answer, be sure that the previous mark is erased completely.

Use your time effectively. Do not spend too much time on questions that are too difficult.

Go on to other questions and come back to the difficult ones later if you have time. It is

not expected that everyone will be able to answer all questions.

Good luck and have fun!

Page 95: 2001 -1995

Should you guess the answers to questions about

which you are not certain?

Since your score on the exam is based on the number of questions you answered correctly minus one-third of the number you answered incorrectly, it is improbable that guessing will improve your score (it is more likely to lower your score). (No points are deducted or awarded for unanswered questions).  However, if you are not sure of the correct answer but have some knowledge of the question and are able to eliminate one or more of the answer choices, then your chance of getting the right answer is improved, and it may be advantageous to answer such a question.

1. All cells have:

a. a cell wall and nucleus.

b. a nucleus and chloroplasts.

c. plastids and lysosomes.

d. a cell membrane and cytoplasm.

e. ribosomes and chloroplasts.

[The correct answer is...]

2. A scientist added a chemical (cyanide) to an animal cell to stop aerobic respiration. Which of

the following is most likely to have been affected by this treatment?

a. Active transport of substances across the plasma membrane

b. Passive transport of substances across the plasma membrane

c. Diffusion of substances across the plasma membrane

d. The size of the ribosomes in the cytoplasm

e. The thickness of lipid bilayers

[The correct answer is...]

3. Which statement is FALSE?

a. If a lysosome bursts, its contents can seriously damage the cytoplasm of a cell.

b. Chloroplasts and mitochondria are each bounded by two membranes.

c. Macromolecules may be taken up into a cell by the process of endocytosis.

Page 96: 2001 -1995

d. Protein molecules that are synthesized by rough endoplasmic reticulum are

commonly modified in the Golgi apparatus.

e. In a hypertonic solution, a cell from a multicellular animal usually will swell

and burst.

[The correct answer is...]

4. If you could suddenly remove all the protein molecules from the plasma membrane of a cell

(without destroying the cell), which of the following would you expect to happen?

a. Transport of all molecules across the plasma membrane would stop.

b. Transport of most ions across the plasma membrane would stop.

c. The amount of cholesterol in the plasma membrane would decrease.

d. Amino acids would rapidly aggregate on the plasma membrane and replace the

missing proteins.

e. Large macromolecules would diffuse out of the cell.

[The correct answer is...]

5. Which label on the diagram of a peanut (shown below) is NOT correct?

a. Cotyledon

b. Hypocotyl

c. Seed coat

d. Meristem

e. Plumule

[The correct answer is...]

6. Which of the following changes would DECREASE the rate of water loss (transpiration) from

the leaves of a sugar maple tree?

a. Light breeze

b. Bright sunshine

c. An afternoon rain shower

d. A plentiful supply of soil water

e. High temperatures during the afternoon

[The correct answer is...]

Page 97: 2001 -1995

7. Deep-water rice is an important crop in southeast Asia where water levels during the rainy season may rise by several metres within a short time. Deep-water rice has the ability to adapt to the rising water by growing quickly, thus keeping its "head" (flower stalk and upper leaves) above water. Examine the four graphs above and identify the hormone(s) that regulate(s) this rapid growth.

a. Ethylene alone

b. Cytokinin alone

c. Gibberellin alone

d. Ethylene and gibberellin

e. Cytokinin and gibberellin

[The correct answer is...]

8. Cactus plants are adapted to the hot and dry deserts of North and South America. Which of the

following characteristics would NOT help cacti to survive the extreme heat and low rainfall?

a. A thick waxy cuticle

b. A stem with a high surface to volume ratio

c. Stomata that are closed during the day

d. Thin leaves with a large surface area

e. Water storage tissue

Page 98: 2001 -1995

[The correct answer is...]

9. During the malting process, starch stored in the barley grain is broken down to sugars. These

sugars provide the energy for anaerobic respiration by yeast cells during the beer making process.

Which of the following observations indicates that the plant hormone, gibberellic acid, is required

for this process?

a. The yeast cells respire aerobically if O2 is bubbled through the brew.

b. Gibberellic acid promotes stem elongation.

c. Amylase enzyme converts starch to sugar.

d. The plant hormone, abscisic acid, prevents barley seed germination.

e. No alcohol is produced if the embryo is removed from the barley grain.

[The correct answer is...]

10. One of Mendel's laws states the following: Units of inheritance which control different aspects

of the phenotype may be inherited independently. Which of the following true statements best

restates this conclusion in modern terms?

a. In the first stage of meiosis, the segregation of one pair of chromosomes does

not affect the segregation of other chromosomes.

b. During fertilization, which sperm combines with which egg is a matter of

chance.

c. In mitosis, there is no difference between the original DNA strand and the

recently synthesized copy.

d. In the first stage of meiosis, chromosomes pair with their homologues.

e. Mutations are random events, which affect each locus independently.

[The correct answer is...]

11. Before beginning a genetic analysis how would you best ensure that you have homozygous

genotypes?

a. Mate close relatives for many generations.

b. Examine each individual very closely for signs of variation.

c. Start with individuals who are all homozygotes for recessive alleles.

d. Start with individuals from different species.

e. Start with individuals which look very different from other individuals.

[The correct answer is...]

Page 99: 2001 -1995

12. In a population of humans, the frequency of a recessive allele causing a genetic disease is 0.01,

or 1%. What proportion of the population would you expect to suffer from the disease?

a. 0.0001

b. 0.001

c. 0.0025

d. 0.01

e. 0.25

[The correct answer is...]

13. In the pedigree shown below, individuals with the solid symbols suffer from a genetic disease

caused by a recessive allele at an autosomal locus. You would counsel the couple marked A and B

that the probability that each of their children will have the disease is:

a. 0

b. 25%

c. 50%

d. 75%

e. 100%

[The correct answer is...]

14. One day, you wake up with a sore throat and a runny nose. Your doctor takes a swab from your

throat, sends it to a lab, and telephones you the next day to say that an antibiotic will not help you

get better. Which of the following is the most likely reason for the doctor's statement?

a. Having waited a day, it is too late to take an antibiotic.

b. You need an antiseptic, not an antibiotic.

c. You need to be vaccinated instead of taking an antibiotic.

d. You are infected by a virus.

e. An antibiotic only eliminates the symptoms of infection and does not cure the

infection itself.

[The correct answer is...]

15. Which statement is FALSE?

Page 100: 2001 -1995

a. Viruses contain only DNA and protein.

b. All bacteria are prokaryotic.

c. All viruses are parasites.

d. Bacteria are involved in nitrogen processing in the soil.

e. Heterotrophic bacteria usually are photosynthetic.

[The correct answer is...]

16. You open your refrigerator to make a sandwich and find that the tomato that you put there 3

weeks ago has turned into a soggy, mushy mess. Your mother tells you that the tomato must have

been infected with a bacterium. Which of the following procedures would be the best way to prove

this?

a. Take a bit of the mushy tomato and analyse it for DNA.

b. Examine the tomato tissue under the electron microscope and look for small

cells with a cell wall, a plasma membrane, and no endoplasmic reticulum.

c. Put a piece of the mushy tomato (using aseptic techniques) on a culture

medium and look for fluffy colonies of micro-organisms.

d. See if the mushy tomato takes up more oxygen than a healthy tomato.

e. Examine the tomato tissue under a light microscope and look for small

polyhedral particles.

[The correct answer is...]

17. How is chyme (the semi-liquid stomach contents) treated in the stomach and small intestine?

a. The compacted chyme is pushed into the small intestine only after the stomach

has completed its action.

b. The chyme has a high pH in the stomach, but that is changed to a low pH in

the intestine.

c. Most of the nutrients are absorbed from the chyme before the chyme is passed

into the small intestine.

d. Small amounts of chyme are continuously released into the small intestine

through the pyloric sphincter.

e. Sucrose and proteins are broken down in the stomach by the enzymes sucrase

and trypsin.

[The correct answer is...]

18. Which statement is FALSE?

Page 101: 2001 -1995

a. The surface area of the absorptive surface of the small intestine of humans is

well over 100 square metres.

b. Your large intestine contains many microorganisms that are beneficial to you

because they produce useful vitamins.

c. In herbivores such as cows, the cellulose of plant cell walls is digested by

cellulase produced by glands in the wall of the rumen.

d. Human saliva contains enzymes that begin the process of breaking down

carbohydrates.

e. The liver produces bile, which helps in the digestion of fats by causing them to

be dispersed as microscopic droplets.

[The correct answer is...]

19. If you arrange the following structures in the order in which they would be passed by an

oxygen molecule during inspiration, which would be third?

a. Bronchiole

b. Trachea

c. Alveolus

d. Epiglottis

e. Pharynx

[The correct answer is...]

20. Which of the following produces negative pressure within your thoracic cavity?

a. Exhalation

b. Contraction of the diaphragm muscles

c. Relaxation of the muscles between the ribs

d. Contraction of the muscles in the wall of the stomach

e. Relaxation of the diaphragm muscles

[The correct answer is...]

21. What normally causes blood to move in the veins back to the heart?

a. Contraction of precapillary sphincters

b. Contraction of nearby skeletal muscles

c. The closing of valves in the veins

d. Gravity

e. Energy stored in elastic fibres in the walls of the veins

Page 102: 2001 -1995

[The correct answer is...]

22. Human males produce all the following hormones EXCEPT:

a. Testosterone

b. Luteinizing hormone

c. Follicle stimulating hormone

d. Gonadotrophins

e. Progesterone

[The correct answer is...]

23. How are the time of ovulation and the onset of menstruation related in the human menstrual

cycle?

a. Both are triggered by high luteinizing hormone "spikes" (sharp increase in

concentration).

b. Ovulation occurs approximately 7 days after the first day of menstruation.

c. Ovulation occurs approximately 14 days before the first day of menstruation.

d. Ovulation is triggered by copulation; menstruation is triggered by hormonal

effect.

e. Ovulation is triggered by hormonal effect; menstruation is triggered by

copulation.

[The correct answer is...]

24. The diagram below is of the adult human heart. The chambers or vessels that carry oxygenated

blood include which of the following?

a. 1 and 2 only

b. 1, 2, and 4

c. 3 only

d. 3, 5, and 6

Page 103: 2001 -1995

e. 5 and 6 only

[The correct answer is...]

25. Which of the following is a "reduction" reaction?

a. Na + H2O --> Na+ + HO- + 1/2H2

b. H2 --> 2H+ + 2e-

c. CH4 + O2 --> CH2O + H2O

d. 2H2 + O2 --> 2H2O

e. Cl2 + 2e- --> 2Cl-

[The correct answer is...]

26. Consider the following: H2O + H2O --> HO- + H3O+. If such an aqueous solution contains 10-6

mol/L of hydronium ion, what is the pH of the solution?

a. 10-8 mol/L

b. -6

c. 6

d. 8

e. 6 mol/L

[The correct answer is...]

27. The requirement for stable hydrogen bonding between base pairs requires that, if one strand of

double-stranded DNA consists of the sequence 3'-ATTCGTAC-5', the complementary sequence

must be:

a. 5'-UAAGCAUG-3'

b. 3'-ATTCGTAC-5' in the reverse direction

c. 3'-TAAGCATG-5'

d. 5'-TAAGCATG-3'

e. 5'-TAAGCATG-3' in the reverse direction

[The correct answer is...]

28. Coupled biochemical reactions are important in determining thermodynamic order within

cells. Consider the following two reactions that occur simultaneously in cells:

Page 104: 2001 -1995

1,3-bisphosphogylcerate --> 3-phosphoglycerate + Pi                     --- (DELTA G° = -49 kJ)ADP + Pi --> ATP                                                                         --- (DELTA G° = +31 kJ)

Which of the following concerning the above partial reactions is FALSE?

a. The energy change of the coupled reaction, or net reaction, is the sum of the

energy changes of the two partial reactions and is DELTA G° = -18 kJ.

b. The coupled reaction is driven toward ATP, or occurs because the sum of the

two free energy changes of the partial reactions is a negative value and yields

energy to drive the reaction.

c. The overall reaction: 1,3-biphosphoglycerate + ADP --> 3-phosphoglycerate +

ATP is endergonic; it requires +18 kJ and thus can not occur spontaneously.

d. The reactions occur during glycolysis.

e. ATP mediates the transfer of energy between exergonic reactions (that

normally have relatively high free energies of phosphate transfer) and endergonic

reactions.

[The correct answer is...]

29. Consider the following:

maleate --> oxaloacetate + 2e- + 2H+

NAD+ + 2e- + 2H+ --> NADH + H+

The overall reaction is: NAD+ + maleate --> NADH + H+ + oxaloacetate

Which statement is FALSE?

a. Maleate is oxidized to oxaloacetate.

b. NAD+ participates in a redox reaction.

c. Maleate is reduced.

d. NAD+ is reduced.

e. NADH can be used as a reducing agent for other synthetic reactions in the cell.

[The correct answer is...]

30. A plant cell was put into a solution of substance A at a concentration of 200 mmol/L. The rate

of uptake into the cell was measured as 5 mmol per minute. When the cell was placed in a solution

of substance A at a concentration of 400 mmol/L, the rate of uptake was 10 mmol per minute.

Page 105: 2001 -1995

When the experiment was repeated with substance B, the rate of uptake was 10 mmol per minute

at both concentrations of the substance. Which of the following provides the best explanation of

the results?

a. Substance A appears to move into the cell by active transport, and B by

diffusion.

b. Substances A and B appear to move into the cell by passive transport.

c. Substances A and B appear to move into the cell by active transport.

d. Substance A appears to move into the cell by diffusion, and B by active

transport.

e. Substance A moves into the cell by active transport, and B by pinocytosis.

[The correct answer is...]

31. Which molecule releases the most energy during cellular respiration?

a. Pyruvic acid

b. Glucose

c. Acetyl coenzyme A

d. Glyceraldehyde

e. Citric acid

[The correct answer is...]

32. With respect to the process of aerobic respiration, which statement is CORRECT?

a. Entropy increases as the process takes place.

b. The reaction violates the second law of thermodynamics because the process is

controlled by enzymes.

c. During the process, entropy is converted into free energy.

d. During the process, the free energy of the system increases.

e. During the process, the free energy remains constant.

[The correct answer is...]

33. Which statement is CORRECT? During aerobic respiration:

a. the energy for oxidative phosphorylation comes from ATP.

b. the energy to make ATP comes from the potential difference in proton

concentration across the inner mitochondrial membrane.

c. ATP is made by the transport of protons through a series of oxidation-reduction

reactions, resulting in the transfer of electrons across the crista membrane of the

Page 106: 2001 -1995

mitochondrion.

d. a mole of glucose may yield from 40 to 50 moles of ATP.

e. glucose is first converted to pyruvic acid in the mitochondrion.

[The correct answer is...]

34. Long distance, competitive, runners are usually small and wiry and run more slowly than

sprinters, who run much shorter distances and generally have a large bulk of muscle. Which of the

following best explains the differences between the two types of runners?

a. Long distance runners run more slowly because lactic acid quickly builds up in

their muscles and causes fatigue. Sprinters do not run for long enough for lactic

acid to build up in their muscles.

b. The large muscles of sprinters increases the oxygen supply to each muscle,

preventing lactic acid from forming.

c. Sprinters do not run for long enough for sufficient lactic acid to build up in

their muscles, therefore they can have large muscles for more power. By being

lighter and running more slowly, long distance runners ensure that their muscles

receive enough oxygen for aerobic respiration.

d. Sprinters run faster because their large muscles have more blood running

through them to stop anaerobic respiration from taking place. Long distance

runners run more slowly because they are using the energy from anaerobic

respiration, which does not produce as much ATP as aerobic respiration.

e. Long distance runners burn more energy than sprinters, which makes it

difficult for them to gain weight.

[The correct answer is...]

Page 107: 2001 -1995

35. Which label on the photograph of a chloroplast shown above is NOT correct?

a. Stroma

b. Thylakoid

c. Chloroplast envelope

d. Location of photophosphorylation

e. Location of Calvin cycle

[The correct answer is...]

36. Which statement about the Calvin cycle is FALSE?

a. 6CO2 + 12H2O --> C6H12O6 + 6O2 + 6H2O

b. The five-carbon sugar ribulose bisphosphate is the acceptor molecule for CO2.

c. Two molecules of phosphoglyceraldehyde (PGAL) combine to form one

molecule of glucose.

d. Phosphoglycerate molecules are phosphorylated using ATP and then reduced

to PGAL by NADPH.

e. ATP from mitochondrial respiration is used to fix carbon and produce

phosphoglyceraldehyde (PGAL).

[The correct answer is...]

37. Transport of the products of photosynthesis is thought to occur by pressure flow through the

sieve tubes of the phloem from a source (such as the leaves where the products are produced) to a

Page 108: 2001 -1995

sink (such as a developing fruit where they are used or stored). Which of the following statements

about phloem transport is FALSE?

a. Water enters the sieve tubes by osmosis.

b. Sieve tubes in a source have a low hydrostatic pressure.

c. Water and solutes move through the sieve tubes along a pressure gradient.

d. Solutes are actively removed from the sieve tubes in the sink area.

e. Solutes are swept along by the bulk flow of water in the sieve tubes.

[The correct answer is...]

38. Using the four graphs above and your knowledge of the action spectrum of photosynthesis and the absorption spectra of chlorophyll a and chlorophyll b, identify the plant pigment responsible for phototropism.

a. Anthocyanin

b. Blue light receptor

c. Carotene

d. Phytochrome

e. Phycocyanin

[The correct answer is...]

Page 109: 2001 -1995

39. Which of the following gives sexual reproduction an advantage over asexual reproduction?

a. It produces more offspring.

b. It ensures the survival of the species.

c. It increases the variation among the offspring of an individual.

d. It preserves parental genotypes.

e. It allows evolution due to sexual selection.

[The correct answer is...]

40. Typically, plant species are adapted for photosynthesis in a specific temperature range. Examine the above graph that shows the relationships between photosynthetic rate and temperature for the following species: alpine tussock grass (a C3 species); wheat from the temperate zone (C3); maize from the subtropics (C4). Which statement about these relationships is FALSE?

a. Wheat is the species that has the highest rate of photosynthesis when

temperatures are around 25°C.

b. Maize shows maximum photosynthetic rates at 38°C.

c. Wheat shows maximum photosynthetic rates at 25°C.

d. In a region where mean temperatures were 10°C, C3 species would have

higher photosynthetic rates than C4 species.

e. In the arctic tundra where mean daily temperatures might be as low as 0°C, the

C3 tussock grass would have the highest rate of photosynthesis.

[The correct answer is...]

41. Normally, genetic information is passed from DNA to RNA to protein. Which recently

discovered process can pass information in the other direction?

a. Digestion by restriction enzymes

b. Polymerase chain reaction

c. DNA repair

d. Reverse transcription

Page 110: 2001 -1995

e. Splicing of introns

[The correct answer is...]

42. The DNA of an organism has cytosine as 20 percent of its bases. What percentage of DNA

bases would be thymine?

a. 0

b. 10

c. 20

d. 30

e. 60

[The correct answer is...]

43. For a gene with the DNA sequence TACCCGGATTCA, and read from left to right, the

anticodon of the transfer RNA that carried the last amino acid would be:

a. TCA

b. UCA

c. AGU

d. ACT

e. AGT

[The correct answer is...]

44. Some of the most effective antibiotics work by blocking the movement of the ribosome along

prokaryotic mRNA. If the antibiotic erythromycin is administered, which of the following

immediate effects would be expected in a bacterial cell?

a. Inhibition of transcription of DNA to RNA

b. Inhibition of translation of RNA to protein

c. Inhibition of DNA replication

d. Inhibition of rRNA transcription only

e. Inhibition of amino acid synthesis

[The correct answer is...]

45. Assume that you are trying to insert a gene from human DNA into a plasmid and someone

gives you a preparation of human DNA cut with restriction endonuclease A. The human gene you

are after has sites on both ends for restriction endonuclease B. You have a plasmid with a single

Page 111: 2001 -1995

site for B, but not for A. Your best strategy would be to:

a. insert the fragments cut with B directly into the plasmid without cutting the

plasmid.

b. cut the plasmid with restriction endonuclease A and insert the fragments cut

with B into the plasmid.

c. cut the plasmid twice with restriction endonuclease B and ligate the two

fragments onto the ends of the human DNA fragments cut with restriction

endonuclease A.

d. cut the human DNA again with the restriction endonuclease B and insert these

fragments into the plasmid cut with the same enzyme.

e. cut the plasmid with restriction endonuclease A and insert the human DNA cut

with A into the plasmid.

[The correct answer is...]

46. Darwin and Wallace convinced most of their contemporaries that evolution had occurred. They

did this primarily by relying on evidence from which area of study?

a. Palaeontology

b. Geographic distribution

c. Behaviour

d. Developmental biology

e. Comparative anatomy

[The correct answer is...]

47. Which of the following observations was NOT important in helping Darwin and Wallace

develop their theory of natural selection?

a. In most species more offspring are produced than can be supported by their

environment.

b. The Earth, and life on Earth, is very old.

c. There is variability in populations.

d. Young tend to resemble their parents.

e. All cells contain DNA which transmits coded information to other cells.

[The correct answer is...]

48. Which of the following would NOT generally affect allele frequencies in a population?

a. Non-random mating

Page 112: 2001 -1995

b. Directional selection

c. Mutation

d. Immigration

e. Emigration

[The correct answer is...]

49. Which of the following would generally reduce the likelihood of speciation?

a. Geographical isolation

b. Genetic variation in populations

c. Natural selection

d. Genetic drift

e. Immigration and emigration

[The correct answer is...]

50. In a population with two alleles for a particular locus, A and a, the frequency of A is 0.6. What

would be the frequency of heterozygotes if the population was in Hardy-Weinberg equilibrium?

a. 0.16

b. 0.24

c. 0.36

d. 0.48

e. 0.64

[The correct answer is...]

51. The human kidney:

a. is responsible for the storage of nutrients such as glycogen.

b. concentrates the urine by actively transporting water out of the filtrate.

c. produces more dilute urine when the collecting ducts become less permeable to

water.

d. responds to antidiuretic hormone by increasing urine output.

e. gets rid of urea from the body by secreting it into the descending arm of the

loop of Henle.

[The correct answer is...]

52. Which of the following is FALSE?

Page 113: 2001 -1995

a. In vertebrate sensory neurons, nerve impulses normally travel both away from

and toward the cell body.

b. The resting potential of a neuron is maintained by membrane "pumps" actively

transporting sodium into and potassium out of the cell.

c. Neurons operate with two main types of electrical signal: slow graded

potentials and fast action potentials.

d. Saltatory conduction involves nerve impulses "jumping" between regions of

the axon where the myelin sheath is missing.

e. The "processing" of information by integration of synaptic activity occurs in

the spinal cord as well as the brain.

[The correct answer is...]

53. Steroid hormones:

a. pass easily through cell membranes to act in the nucleus.

b. include testosterone, estrogen, and growth hormone.

c. are only produced in the pituitary gland.

d. stimulate liver cells to convert glucose to glycogen.

e. are made of short chains of amino acids.

[The correct answer is...]

54. Insulin:

a. was discovered by Banting and Best in the 1970s; both received the Nobel

prize for their work.

b. acts on the liver to cause the release of glucose when glucose is needed during

exercise.

c. was originally difficult to obtain because it is destroyed by protein-digesting

enzymes from the pancreas.

d. causes cell membranes to become impermeable to glucose.

e. can cause diabetic coma by raising blood sugar levels higher than normal.

[The correct answer is...]

55. Which sequence best describes a simple reflex arc such as the knee-jerk reflex?

a. sensory neuron --> interneuron --> motor neuron --> effector cell

b. sensory neuron --> interneuron --> interneuron --> motor neuron

c. sensory neuron --> motor neuron --> interneuron

Page 114: 2001 -1995

d. sensory neuron --> effector cell --> motor neuron

e. sensory neuron --> motor neuron --> effector cell

[The correct answer is...]

56. An excess supply of which of the following nutrients is the most common cause of

eutrophication in freshwater lakes in Canada?

a. Phosphorus

b. Calcium

c. Sulfur

d. Potassium

e. Nitrogen

[The correct answer is...]

57. Because of interest in the global carbon (C) cycle and the greenhouse effect, ecologists are

estimating how much C is in each biome and whether the sizes of these C pools are changing. The

organic matter (humus and dead plant tissues) in the soils of the boreal forest biome forms one of

the largest pools of C in the biosphere. On average, boreal forest soils contain 15,000 g C m -2, a

higher value than occurs in other types of forest. However, the primary production of the boreal

forest is only 350 g C m-2 yr-1 on average, a low value compared to other types of forests. This

pattern occurs primarily because:

a. high rates of herbivory in the boreal forest lead to low levels of forest

productivity.

b. the slow weathering of bedrock under boreal forest soils releases large

quantities of C into the soil. This C would be lost in hotter climates.

c. low rates of herbivory in the boreal forest mean that most leaves, branches, and

roots are added to the soil when they die. Temperate and tropical forests, in

contrast, have high rates of herbivory.

d. the cold, wet soils of the boreal forest restrict decomposition more than they

restrict the photosynthesis of plants.

e. periodic fires destroy large areas of boreal forest and keep primary

productivity low, while leaving most of the soil C intact.

[The correct answer is...]

58. Many of the world's fisheries are severely depleted. The goal of sustainable harvesting in a

fishery is to harvest only a constant proportion of the total population, for instance 30%.

Unfortunately, the harvest size has often gone up or stayed constant even as the total population

Page 115: 2001 -1995

size has crashed. Here is simple model of a fishery:

H = N x E x G

H = harvest size (number of fish caught)N = total population size (number of fish)E = the harvest effort (the number of days spent fishing times the number of boats fishing; in other words, the units of E are boat-days)G = efficiency (the proportion of N caught per boat per day)Which management approach is most sustainable and will NOT cause the population to crash in the long run?

a. As N decreases, increase G by using either sonar to find the fish more easily,

nets with smaller mesh sizes, or other advances in technology.

b. As N decreases, increase E by allowing more boats to fish or more days of

fishing.

c. As N decreases, increase H.

d. As N decreases, set a level for H and keep it fixed at that level.

e. As N decreases, either E or G can change, but their product must remain

constant.

[The correct answer is...]

59. We have introduced an animal species to an island where it undergoes exponential growth in

this new unexploited, resource-rich environment. The initial population size (year 0) is 20

individuals (point shown on graphs). After 4 years the population size is 80 individuals. What is

the population size after 10 years, assuming that exponential growth has continued?

a. The population will be at its carrying capacity, but too little information about

the biology of the species is given to determine what that value is.

b. About 140 individuals

c. About 170 individuals

d. About 400 individuals

e. About 640 individuals

Page 116: 2001 -1995

[The correct answer is...]

60. In freshwater lakes, the dissolved nutrients are mixed and redistributed primarily by:

a. fish.

b. high and low tides.

c. the spring and fall turnover.

d. the thermocline.

e. wave action along shorelines.

[The correct answer is...]

The University of Toronto Ontario Biology Competition

1996 Examination

Answer Key

1. d 2. a 3. e 4. b 5. a

6. c 7. c 8. b,d 9. e 10. a

11. a,c 12. a 13. c 14. d 15. a,e

16. b 17. d 18. c 19. b 20. b

21. b 22. e 23. c 24. d 25. e

26. c 27. d 28. c 29. c 30. d

31. b 32. a 33. b 34. c 35. a

36. a,e 37. b 38. d 39. c 40. a

41. d 42. d 43. b 44. b 45. d

46. b 47. e 48. a 49. e 50. d

51. c 52. b 53. a 54. c 55. e

56. a 57. d 58. e 59. e 60. c

Note:  In four questions two responses were given credit as being correct answers

(i.e., if a student selected either of these responses it was recorded as a correct

response): in questions 8, 11, and 15 two responses were deemed to be correct

Page 117: 2001 -1995

(this was unintentional); response (a) of question 36 had a typographical error

(O6, not O26) in the original version of the test which made this response correct,

in addition to response (e).

The University of Toronto Ontario Biology Competition

1995 Examination Time: 90 minutes

Number of Questions: 60

Answer Key

General Instructions

Do not open this booklet until you are instructed to do so.

Print your name at the top of this booklet.

Indicate all of your answers to the questions on the separate Response Form. No credit

will be given for anything written in this booklet, but you may use the booklet for notes or

rough work. No additional time will be given after the exam to transfer your answers to

the Response Form.

After you have decided which of the suggested answers is best, COMPLETELY fill in the

corresponding circle on the Response Form. Give only one answer to each question. If

you change an answer, be sure that the previous mark is erased completely.

Use your time effectively. Do not spend too much time on questions that are too difficult.

Go on to other questions and come back to the difficult ones later if you have time. It is

not expected that everyone will be able to answer all questions.

Good luck and have fun!

Should you guess the answers to questions about

which you are not certain?

Since your score on the exam is based on the number of questions you answered correctly minus one-fourth of the number you answered incorrectly, it is improbable that guessing will improve

Page 118: 2001 -1995

your score (it is more likely to lower your score). (No points are deducted or awarded for unanswered questions).  However, if you are not sure of the correct answer but have some knowledge of the question and are able to eliminate one or more of the answer choices, then your chance of getting the right answer is improved, and it may be advantageous to answer such a question.

1. If a plant cell has its cell wall removed and is placed in a hypotonic solution, what will happen to the cell?

a. The cell will already be dead because plant cells cannot survive without a cell

wall.

b. The cell will expand and eventually burst.

c. The cell will shrink because the vacuole gets smaller.

d. Nothing obvious would happen.

e. The nucleus will burst, but the cell will stay intact.

[The correct answer is...]

2. Under a light microscope, tiny dots can be seen in the cytoplasm of living plant and animal

cells. These are likely to be:

a. chloroplasts.

b. ribosomes.

c. mitochondria.

d. nuclei.

e. chromosomes.

[The correct answer is...]

3. Which statement is FALSE?

a. During the process of exocytosis, the lumen (inner) surface of a secretory

vesicle becomes the inside of the plasma (cell) membrane.

b. Cilia, microtubules, and flagella are all associated with movement in cells.

c. A function of the nucleus is duplicating the genes for cell division.

d. Proteins that are to be secreted by the cell are generally synthesized by

membrane-bound ribosomes.

e. Active transport involves movement of molecules into a cell against their

concentration gradient.

Page 119: 2001 -1995

[The correct answer is...]

4. A scientist who worked on a single-celled green plant (an alga) treated it with ultraviolet light

and produced mutant cells that did not contain chloroplasts. To keep these cells alive, the scientist

had to:

a. provide them with high light intensity.

b. keep them in the dark.

c. provide them with a sugar solution.

d. give them chlorophyll.

e. increase the temperature.

[The correct answer is...]

5. Cotyledons are the first leaves produced by the embryos of vascular plants. Which of the

following is NOT a physiological function of cotyledons?

a. Absorption of carbohydrates from the endosperm.

b. Photosynthesis.

c. Storage of proteins.

d. Storage of lipids.

e. Vegetative propagation.

[The correct answer is...]

6. The vessel elements of the xylem transport water and nutrients under negative pressure. Which

of the following is NOT characteristic of vessel elements?

a. Cells are elongate in shape.

b. Thickened rigid secondary walls are present.

c. The end walls are lysed, forming a perforation.

d. Wide-diameter pores are present at the ends of the cell.

e. The nucleus and cytoplasm undergo autolysis.

[The correct answer is...]

7. The pressure flow hypothesis for the mechanism of phloem transport was developed over 60

years ago by the German plant physiologist Ernst Muench and still remains the "front runner" as

an explanation of how the plant moves carbohydrates over long distances. Which of the following

observations provides direct support for Muench's hypothesis?

Page 120: 2001 -1995

a. Transpiration occurs from open stomata on leaves and stems during the day.

b. Turgor pressure in the sieve tubes of a fully expanded tomato leaf is higher

than the turgor in the sieve tubes of a developing tomato fruit.

c. Vessels transport water under negative pressure.

d. Carbohydrates move within the plant from sink to source.

e. The osmotic potential of the sieve tubes in a root tip is more negative than that

of the sieve tubes in the leaf.

[The correct answer is...]

8. Charles Darwin and his son Francis experimented with phototropism of grass seedlings by

placing a metal foil "blindfold" over different parts of the seedling's coleoptile. A simplified

version of their results is shown below. Which of the following statements best explains their

results?

a. The light signal is perceived a few millimetres below the tip, and these cells

cause the coleoptile to grow toward the light.

b. Both the seedling root and coleoptile perceive and respond to light in the same

manner.

c. A chemical messenger must travel from the base of the coleoptile to the tip.

d. The light signal is perceived at the tip of the coleoptile, but the growth

response occurs a few millimetres below the tip.

e. The coleoptile bends because cells on the side toward the light grow faster than

those on the shaded side.

[The correct answer is...]

9. In 1925, the Japanese biologist Eiichi Kurosawa showed that the bakanae ("foolish seedling")

disease of rice was caused by a substance produced by the fungus Gibberella fujikuroi. The

substance was named gibberellin and was shown to cause elongation of stem tissues. Which of the

Page 121: 2001 -1995

following statements about gibberellin is FALSE?

a. Pea plants carrying the dwarf mutation would be expected to have higher

levels of gibberellin in their stems than normal plants.

b. If gibberellin is applied to the stems of dwarf pea plants, the stems elongate so

that plant reach normal height.

c. Dwarf pea plants have a mutation in the gibberellin biosynthetic pathway.

d. Normal pea plants respond to gibberellin by growing even taller.

e. The gibberellin produced by the fungus caused normal healthy rice plants to

become unusually tall.

[The correct answer is...]

10. Which of the following statements about Mendelian genetics is FALSE?

a. Alternate forms of genes are called alleles.

b. A locus is a gene's location on its chromosome.

c. Only two alleles can exist for a given gene.

d. A genotype is a description of the alleles that represent an individual's genes.

e. Individuals with the same phenotype can have different genotypes.

[The correct answer is...]

11. In corn, the trait for tall plants (T) is dominant to the trait for dwarf plants (t) and the trait for

coloured kernels (C) is dominant to the trait for white kernels (c). In a particular cross of corn

plants, the probability of an offspring being tall is 0.5 and the probability of a kernel being

coloured is 0.75. Which of the following most probably represents the parental genotypes?

a. TtCc x TtCc

b. TtCc x ttCc

c. TtCc x ttcc

d. TTCc x ttCc

e. TTCc x TtCC

[The correct answer is...]

12. A cell cycle consists of:

a. mitosis and meiosis.

b. G1, the S phase, and G2.

c. prophase, metaphase, anaphase, and telophase.

d. interphase and mitosis.

Page 122: 2001 -1995

e. meiosis and fertilization.

[The correct answer is...]

13. How do cells at the completion of meiosis compare with the diploid cell from which they were

derived?

a. They have twice the amount of cytoplasm and half the amount of DNA.

b. They have half the number of chromosomes and half the amount of DNA.

c. They have the same number of chromosomes and half the amount of DNA.

d. They have the same number of chromosomes and the same amount of DNA.

e. They have half the amount of cytoplasm and twice the amount of DNA.

[The correct answer is...]

14. Which statement is CORRECT?

a. Viruses can grow in food in your refrigerator.

b. All viruses are dangerous to humans.

c. Viruses are prokaryotic organisms.

d. Viruses can take over control of eukaryotic cells.

e. Viruses are slightly larger than bacteria.

[The correct answer is...]

15. When a doctor gives you an antibiotic when you are sick, he/she always tells you to keep

taking the antibiotic until it is all finished. The reason he/she tells you this is because:

a. it is wasteful not to finish all the antibiotic.

b. viruses require high doses of antibiotics to kill them.

c. allowing the more antibiotic-tolerant bacteria to survive may encourage a

population of antibiotic-resistant bacteria to evolve.

d. a long period of antibiotic use is required to prevent secondary virus infection.

e. bacteria have a cell wall and thus it takes a long time for antibiotics to work.

[The correct answer is...]

16. Which statement is FALSE? Bacteria are involved in:

a. animal diseases.

b. decomposing dead organic matter.

Page 123: 2001 -1995

c. nitrogen processing in the soil.

d. food digestion in animals.

e. alcohol production in beer.

[The correct answer is...]

17. The secretion of a less than normal amount of the hormone gastrin would have which effect?

a. Stomach pH would decrease.

b. Protein digestion in the stomach would decrease.

c. Carbohydrate digestion in the stomach would increase.

d. Secretion of alkaline mucus in the stomach lining would increase.

e. The sensation of heartburn would begin.

[The correct answer is...]

18. Food passing beyond the back teeth signals all of the following EXCEPT:

a. churning of the stomach contents.

b. elevation of the palate to seal off the nasal cavity.

c. folding of the epiglottis over the trachea.

d. pressure against the pharynx initiating swallowing.

e. signals from the swallowing centre inhibit breathing.

[The correct answer is...]

19. Gas exchange in animals always involves:

a. cellular respiration.

b. breathing movements.

c. neural control of exchange.

d. diffusion across membranes.

e. active transport of gases.

[The correct answer is...]

20. Which statement explains why fish spend a lot of energy removing oxygen from water?

a. They have to pump large volumes of water through their gills to keep their

respiratory membranes moist.

b. The CO2 content of their tissues is much higher than that of terrestrial animals.

Page 124: 2001 -1995

c. Their gills are covered with protective plates which make it more difficult for

them to process the oxygen from air.

d. They have to pump large volumes of water out of their gills because of water's

high oxygen content.

e. They have to pump large volumes of water through their gills because of

water's low oxygen content.

[The correct answer is...]

21. In the cardiac cycle, blood pressure is at a maximum when:

a. the atria are contracting during systole.

b. the atria are contracting during diastole.

c. the ventricles are contracting during systole.

d. the ventricles are relaxing during systole.

e. the ventricles are relaxing during diastole.

[The correct answer is...]

22. Where is blood pressure lowest during diastole?

a. arteries

b. ventricles

c. capillaries

d. venules

e. veins

[The correct answer is...]

23. The series of mitotic divisions a zygote goes through immediately after fertilization is called:

a. cleavage.

b. blastomere.

c. blastula.

d. blastocoel.

e. blastodisk.

[The correct answer is...]

24. The secretory phase of the menstrual cycle:

Page 125: 2001 -1995

a. is associated with dropping levels of estrogen and progesterone.

b. is when the endometrium begins to degenerate and menstrual flow occurs.

c. corresponds with the follicular phase of the ovarian cycle.

d. is the beginning of the menstrual flow.

e. corresponds with the luteal phase of the ovarian cycle.

[The correct answer is...]

25. The functional group written as -COOH is called the:

a. hydroxyl group.

b. carbonyl group.

c. amino group.

d. ketone group.

e. carboxyl group.

[The correct answer is...]

26. Which of the following statements is FALSE?

a. In an exergonic reaction, energy released as reactants with a relatively high

energy content form products with a lower energy content.

b. Energy is not required for an endergonic reaction as reactants with a relatively

low energy content are promoted to products with a higher energy content.

c. The hydrolysis of ATP, yielding approximately -7.3 kcal/mole, ADP, and P i, is

coupled with the formation of glucose phosphate that requires +4 kcal/mole.

d. The principle of conservation of energy is called the First Law of

Thermodynamics.

e. Energy-releasing reactions require an amount of energy equal to the activation

energy to allow the reaction to proceed.

[The correct answer is...]

27. Which of the following is a redox reaction?

a. O2 + 4 H+ + 4 e- --> 2 H2O

b. H2 --> 2 H+ + 2 e-

c. 2 H2 + O2 --> 2 H2O

d. Na --> Na+ + e-

e. Cl2 + 2 e- --> 2 Cl-

[The correct answer is...]

Page 126: 2001 -1995

28. Which of the following statements is FALSE?

a. The partial charges on the atoms in water are due to bond polarization.

b. The bond formed between the hydrogen of one molecule of water and the

oxygen of another is called a covalent bond.

c. Cations are positively-charged ions that have given up one or more electrons

from the valence shell.

d. Ionic bonds are involved in the formation of sodium chloride.

e. The hydrophobic effect can be viewed as a process that minimizes the surface

contact between non-polar molecules and polar molecules.

[The correct answer is...]

29. The chemical structure shown below is:

a. a triphosphopeptide.

b. a ribosugar.

c. deoxyadenosine triphosphate.

d. the nitrogenous base, adenine.

e. deoxythymidine triphosphate.

[The correct answer is...]

30. The reaction below represents:

Glucose + 2 ADP + 2 Pi + 2 NAD+ --> 2 pyruvate + 2 ATP + 2 NADH + 2 H+ + 2 H2O

a. cellular respiration.

b. the citric acid cycle.

c. pyruvate oxidation.

Page 127: 2001 -1995

d. glycolysis.

e. substrate-level phosphorylation.

[The correct answer is...]

31. Which of the following produces the most ATP per gram?

a. fats

b. glucose

c. proteins

d. glycogen or starch

e. carbohydrates

[The correct answer is...]

32. The enzymes of the citric acid (Krebs) cycle are located in the:

a. cytoplasm.

b. cristae of the mitochondrion.

c. matrix of the mitochondrion.

d. endoplasmic reticulum.

e. lysosome.

[The correct answer is...]

33. An excess of positive charge from hydrogen ions builds up:

a. in the cytoplasm.

b. in the matrix of the mitochondrion.

c. in the endoplasmic reticulum.

d. in the space between the two mitochondrial membranes.

e. just inside the plasma membrane.

[The correct answer is...]

34. Which of the following is NOT found in the mitochondrion?

a. acetyl-coenzyme A

b. citric acid

c. NADH

d. fatty acids

Page 128: 2001 -1995

e. lactic acid

[The correct answer is...]

35. Most of the water lost by the plant through transpiration exits through the stomatal pores.

Which of the following statements about transpiration and stomatal functioning is FALSE?

a. Stomata normally open in response to a shift from dark to light conditions.

b. When the turgor pressure of guard cells increases, the stomatal aperture opens.

c. With the shift from dark to light, the osmotic potential becomes less negative,

resulting in stomatal closure.

d. The turgor pressure of guard cells increases after the osmotic potential

becomes more negative.

e. A light-activated proton pumping system results in an influx of K+ ions into the

guard cells.

[The correct answer is...]

36. During the light-dependent reactions of photosynthesis (photophosphorylation), light energy is

converted to chemical potential energy through the process of chemiosmosis in the chloroplasts.

Which of the following statements about this process is FALSE?

a. The electron carriers of photophosphorylation are located in the thylakoid

membranes of the chloroplasts.

b. During photophosphorylation, the chloroplast stroma becomes more acidic

than the interior of the thylakoid membranes.

c. Protons diffuse through protein channels which are ATP synthetase molecules.

d. ATP is synthesized from ADP and Pi on the stroma side of the thylakoid

membranes in the chloroplast.

e. During photophosphorylation, water ionizes to form H+ and OH- ions, yielding

an electron to Photosystem II.

[The correct answer is...]

37. Which of the following statements about the Calvin cycle are FALSE?

a. The first stable product of the cycle is 3-phosphoglycerate.

b. The reducing power of NADPH is used to reduce 3-phosphoglycerate to

glyceraldehyde 3-phosphate (PGAL).

c. The carboxylation of the 5-carbon sugar ribulose 1,5 bisphosphate (RuBP) is

catalyzed by the rubisco enzyme (ribulose bisphosphate carboxylase/oxygenase).

Page 129: 2001 -1995

d. Light is not required for the operation of the Calvin cycle.

e. Two molecules of ATP are synthesized for each "turn" of the cycle.

[The correct answer is...]

38. Which of the following labels in the diagram below of a leaf cross-section is NOT correct?

a. epidermal cell

b. palisade mesophyll cell

c. intercellular air space

d. spongy mesophyll cell

e. xylem cell

[The correct answer is...]

39. Increased levels of atmospheric CO2 result in the "greenhouse effect" and thus global climatic

warming. The graph below shows the effect of these changes on photosynthesis. Which of the

following statements about these processes is FALSE?

Page 130: 2001 -1995

a. For most plants, temperatures over 40°C reduce photosynthesis.

b. At temperatures below 20°C, high atmospheric CO2 concentrations reduce

photosynthesis to values below those seen for plants growing in normal CO2

concentrations.

c. One result of increased levels of atmospheric CO2 will be to increase rates of

photosynthesis in most plants.

d. At high CO2 concentrations, the highest rates of photosynthesis are measured

at temperatures between 30° and 35°C.

e. CO2 concentration is normally rate-limiting for photosynthesis.

[The correct answer is...]

40. The two strands of the DNA double helix are held together by:

a. hydrogen bonds.

b. C=C double bonds.

c. hydrophobic bonds.

d. peptide bonds.

e. phosphodiester bonds.

[The correct answer is...]

41. During DNA replication:

a. one parental strand must be degraded to allow the other strand to be copied.

b. the parental strands come back together after the passage of the replication

Page 131: 2001 -1995

fork.

c. origins of replication always give rise to single replication forks.

d. two replication forks diverge from each origin but one always lags behind the

other.

e. the parental strands must separate so that both can be copied.

[The correct answer is...]

42.

GTAGTAGGT

What would be the sequence of bases of an mRNA molecule that was transcribed from the sequence of DNA bases shown above?

a. GTAGTAGGT

b. CAUCAUCCA

c. UCGUCGUUC

d. AUGAUGAAU

e. CATCATCCA

[The correct answer is...]

43. Choose the answer that has the following events of protein synthesis in the proper sequence.

i. An aminoacyl-tRNA binds to the A site.

ii. A peptide bond forms.

iii. tRNA leaves the P site and the P site remains vacant.

iv. Small ribosomal subunit associates with mRNA.

v. Translocation of the tRNA to the P site.

a. i, iii, ii, iv, v

b. ii, iv, v, i, iii

c. iv, i, ii, v, iii

d. iv, i, iii, ii, v

e. v, iv, iii, ii, i

[The correct answer is...]

44. A frameshift mutation could result from:

a. a base insertion only.

b. a base deletion only.

Page 132: 2001 -1995

c. a base substitution only.

d. deletion of three consecutive bases.

e. either an insertion or a deletion of a base.

[The correct answer is...]

45. Restriction enzymes are used in recombinant DNA technology to:

a. cut large DNA molecules at sequence-specific sites.

b. carry foreign genes along with viral DNA into the host cell.

c. join DNA fragments.

d. clone DNA fragments.

e. carry viruses along with plasmid DNA into the host cell.

[The correct answer is...]

46. Which of the following was NOT part of Darwin's explanation of natural selection?

a. Organisms commonly produce more offspring than can possibly survive.

b. Variations exist within each species.

c. Members of a species compete with each other for food and space.

d. Genes are passed on from parent to offspring.

e. The offspring that are most fit possess traits which are best suited to the

environment.

[The correct answer is...]

47. In a population that is in Hardy-Weinberg equilibrium, the frequency of a recessive allele for a

certain hereditary trait is 0.20. What percentage of the individuals in the next generation would be

expected to show the dominant trait?

a. 8%

b. 16%

c. 32%

d. 64%

e. 96%

[The correct answer is...]

48. Which of the following statements best describes the effect of genetic drift on the gene

frequencies of a population?

Page 133: 2001 -1995

a. Genes enter a population through immigration, thus changing gene

frequencies.

b. Genes leave a population through emigration, thus changing gene frequencies.

c. Chance alone can cause significant changes in gene frequencies of small

populations.

d. Mutations over time cause gene frequencies to change.

e. Selection against one allele causes gene frequencies to change.

[The correct answer is...]

49. If the fossil record has few or no intermediate forms, if there are long periods in which the

fossils underwent no morphological change, and if new forms arose very quickly, then evolution

of these new forms would be best described as:

a. punctuated equilibrium.

b. adaptive radiation.

c. gradualism.

d. convergent evolution.

e. due to stabilizing selection.

[The correct answer is...]

50. Which of the following are necessary for evolution by natural selection to take place?

i. Offspring resemble their parents more than other individuals in the population.

ii. Differences among individuals exist and lead to different numbers of

successful offspring being produced.

iii. Individuals adjust their development depending on the environment.

iv. Every individual has a desire to have many offspring.

v. Populations tend to grow faster than their food supplies.

a. i and ii

b. i and v

c. ii, iii, and iv

d. iii and v

e. All of the above.

[The correct answer is...]

51. In regulatory systems, the phenomenon of negative feedback:

a. anticipates an error in the system.

Page 134: 2001 -1995

b. increases the error signal in the system.

c. increases the pace of the system.

d. provides a means of control.

e. decreases the pace of the system.

[The correct answer is...]

52. The endocrine system includes all hormone-producing tissues that:

a. are organized into discrete glands.

b. can be stimulated by the nervous system.

c. have sacs for hormone storage before release.

d. have hormone receptors on their cell surfaces.

e. secrete their products into the blood.

[The correct answer is...]

53. The general functions of the nervous system include which of the following?

i. integration

ii. motor output

iii. sensory input

a. i only

b. ii only

c. iii only

d. i and ii only

e. i, ii, and iii only

[The correct answer is...]

54. What effect does insulin have in a normal human body?

a. It mediates release of glucose by liver cells.

b. It stimulates formation of glycogen in the liver.

c. It stimulates conversion of glycogen to glucose.

d. It increases absorption of glucose in the small intestine.

e. It counteracts the effects of vitamin deficiencies.

[The correct answer is...]

Page 135: 2001 -1995

55. The region of the brain that integrates visceral activities, body temperature, and heartbeat is

the:

a. medulla oblongata.

b. hypothalamus.

c. cerebrum.

d. cerebellum.

e. corpus callosum.

[The correct answer is...]

56. The traditional concept of succession includes the idea of an equilibrium state called a climax

community. Ecologists now think there may be no such thing as a climax community because:

a. disturbance is ongoing in ecosystems.

b. all organisms eventually die.

c. species diversity generally increases.

d. extinction increases in stable environments.

e. each succession is different from others.

[The correct answer is...]

57. In the human-induced process called acid precipitation, the main biogeochemical cycles that

are altered are the __________ cycles and one effect in lakes is to __________ populations of

nitrifying bacteria.

a. phosphorus and nitrogen, increase

b. phosphorus and nitrogen, decrease

c. nitrogen and sulfur, decrease

d. nitrogen and sulfur, increase

e. phosphorus and sulfur, decrease

[The correct answer is...]

58. Which of the following statements about food chains and energy flow through ecosystems is

FALSE?

a. A single organism can feed at several trophic levels.

b. Detritivores feed at all trophic levels except the producer level.

c. The lower the trophic level at which an organism feeds, the more energy

available.

d. Food webs include two or more food chains.

Page 136: 2001 -1995

e. All organisms that are not producers are consumers.

[The correct answer is...]

59. The actual rate of growth of a population is the difference between the:

a. number of adults and the number of newborn.

b. numbers of breeding and non-breeding individuals.

c. size last year and the size this year.

d. birth rate and death rate.

e. size this year and the size next year.

[The correct answer is...]

60. Which of the following has most probably had the greatest impact on human population

growth over the past 10,000 years?

a. The move from a nomadic way of life to growing and cultivating crops in a

fixed location which occurred approximately 8,000 to 10,000 years ago.

b. The improvements to housing, sanitation, and health that occurred during the

Industrial Revolution.

c. The control of human diseases, such as small pox, measles, and malaria.

d. The increase in food production through the use of fertilizers, mechanization,

plant and animal breeding, pest control, and irrigation.

e. development of technologies to transport people and materials efficiently and

to process and communicate enormous amounts of information.

[The correct answer is...]

The University of Toronto Ontario Biology Competition

1995 Examination

Answer Key

1. b 2. c 3. a 4. c 5. e

6. d 7. b 8. d 9. a 10. c

Page 137: 2001 -1995

11. b 12. d 13. b 14. d 15. c

16. e 17. b 18. a 19. d 20. e

21. c 22. b 23. a 24. e 25. e

26. b 27. c 28. b 29. c 30. d

31. a 32. c 33. d 34. e 35. c

36. b 37. e 38. d 39. b 40. a

41. e 42. b 43. c 44. e 45. a

46. d 47. e 48. c 49. a 50. a

51. d 52. e 53. e 54. b 55. b

56. a 57. c 58. b 59. d 60. c

卑诗省、育空、主机

课程指南

11 和 12 年级生物

11 级 (20 生物学 )

1 -调整和变化 2 -微生物:病毒 3 -微生物:王国 Monera

4 -微生物:KingdomProtista

5 --Mycology

6 植物生物学:绿藻类、Mosses,蕨类 7 植物生物学:裸子植物 8 植物生物学:angiosperms

9 -动物生物学:Porifera,Cnidaria

10 -动物生物学:Platyhelminthes,Ne

matoda,

Annelida

 

 

 

 

t 他 "知识目标" 为使各单位在下

面. 这些目标是从 明和 11 年级的

学习成果(1996)12 生物学、 (教

育部、技能培训、卑诗省).

全部版本的 11 和 12 年级的课程

Page 138: 2001 -1995

11 -动物生物学:Mollusca,Echinoder

mata

12 -动物生物学:Arthropoda

13 -动物生物学:Chordata-

subphylumVertebrata

14 -生态学

12 年级 ( 生物 30)

细胞生物学 一 . 细胞结构 B . 细胞化合物 C . -分子生物学 D . DNA

e . --蛋白质合成 F . 癌 G . 运输跨越细胞膜 H . -酶

人类生物学 一 . -消化系统 J . -循环系统:血液循环和 K . -循环系统:心脏结构及功能 L . -呼吸器官 M . -神经系统:Neuron,冲动的一代,

与反射弧 N . -神经系统:神经师 系统和大脑 O . -泌尿系统 P . -生殖系统

可在文件:

http://www.bced.gov.bc.ca/irp/biol

ogy/apa11.htm

以及 http://www.bced.gov.bc.ca/irp/biol

ogy/apa12.htm

进一步资料:

卑诗省教育部

教育和培训部扩展

教育部培训和育空

11 级 (20 生物学 )

Page 139: 2001 -1995

一、适应变化

预计学生会:

描述 DNA 的基本结构

在 DNA 鉴定的作用演变

解释变化中的作用和有性生殖的发展

自然选择过程描述

建议的情况下,人口 allelic 频率可以改变,包括基因漂移,不同移民突变、自然选择

区分,举例说明趋同,差异,speciation

与此相反,逐步改变模式与平衡模式的演进笑声

在发展中的作用确定灭绝

2 微生物 (电脑)

预计学生会:

描述了病毒的基本结构

用于评估证据,把病毒生物或非生物

与此相反的,比较 lyticlysogenic 周期

描述人体的基本路线抵御病毒攻击

举例说明如何减少病毒的感染机会病

确定病毒的具体实例,让

评估病毒对人类健康的影响

3 微生物 (王国 Monera)

预计学生会:

描述基本结构的细胞 prokaryotic

审查委员王国 Monera 和描述特征,把它们

Page 140: 2001 -1995

用实例 Moneran 多样性方面:形式、分配、精虫、生态作用、营养与人类疾病

区分发酵,有氧呼吸,光合作用,Monerans

对比的方法,使他们的细菌分解食物和寄生虫

结果显示,技术准备动盘

显示正确使用复合显微镜

效益评估各种抗生素、消毒剂和抗菌剂对细菌文化

解释适应过程,成为细菌对抗生素抗药性

举例说明有益细菌的作用

4 微生物 (KingdomProtista)

预计学生会:

审查委员王国 Protista,描述特征,把它们

编写湿发动幻灯片

浮游植物和浮游动物的区别观察生活 protists

与此相反,一个细胞 prokaryotic(MONERAN)细胞的遗传(protist)

表明了病原 protist知识的生命周期可以用来控制其蔓延

结构调整涉及到不同的角色 protists 食物链

5mycology

预计学生会:

英国真菌研究的描述和特点,把它们

结果显示,技术准备文化

用科学的方法设计实验

评估显示,适合各种真菌的生长条件

关于修改<真菌的生态系统中的作用不同

6 植物生理 (绿藻类,Mosses、蕨类)

Page 141: 2001 -1995

预计学生会:

绿色藻类研究、松下、蕨类、描述特征,把每个

解剖表明正确使用显微镜

解释比较利益代

采用单细胞的例子,殖民地,结合绿色藻类的多样性说明

描述绿色藻类生态作用,空气、蕨类

描述空气作用的植物为先驱

与此相反,蕨类、苔藓如何适应陆地环境

7 植物生理 (裸子植物)

预计学生会:

描述特征,把裸子植物

说明裸子植物适合生存的环境,对于土地:世代交替,针、种子、花粉、细胞治疗

解释的作用是增加中小学 meristems

评估经济和生态重要性裸子植物

8 植物生物学 (angiosperms)

预计学生会:

angiosperms研究和描述特征,把它们

与此相反的方式比较 angiosperms 裸子植物适应土地和环境

样品和使用区分 monocotsdicots

9 动物生物学 (Porifera,Cnidaria)

Page 142: 2001 -1995

预计学生会:

PhylumPorifera 的研究和描述,PhylumCnidaria 特点,把每个

解释过程提供了过滤海绵

了解生态示范作用和海绵 cnidarians

与此相反,斯巴达对息肉、结构、总功能、精虫

建议的优势,形成了 grallator 生命周期的 cnidarian

结合进化的意义解释(cnidarian)殖民地与生命形态(Poriferan)

10 动物生理 (Platyhelminthes,Nematoda,Annelida)

预计学生会:

研究成员 PhylumPlatyhelminthes,PhylumNematoda,PhylumAnnelida,描述特征,把每个

安全技术及示范正确剖析

相反的结构特征 platyhelminthes、昆虫病原线虫、节肢动物

检查身体,并解释变革,必须有士气,成为寄生

生态知识展示 platyhelminthes 的作用,昆虫病原线虫、节肢动物

评价一个成功的寄生虫的特点

比较 platyhelminthes、昆虫病原线虫、节肢动物进化方面变化

展示人类疾病的知识,是由非昆虫病原线虫分

11 动物生理 (Mollusca,Echinodermata)

预计学生会:

PhylumMollusca 的研究和描述,PhylumEchinodermata 特点,把每个

与此相反,两个或两个以上的成员类班

展示的知识和改编种类

展示知识的各种生态作用及种类

Page 143: 2001 -1995

12 动物生理 (Arthropoda)

预计学生会:

PhylumArthropoda 的研究和描述特征,把它们

对照班成员有两个或两个以上的节肢动物

展示的知识,以适应地球环境的节肢动物

升值表现的重要节肢动物的生态和经济多元化

13 动物生理 (Chordata-subphylumVertebrata)

预计学生会:

审查委员脊椎动物与描述特征,把它们

对照班成员有两个或两个以上的脊椎动物

相反的结构和功能的装置的 endoskeleton

展示知识的各种生态作用脊椎动物

14 -生态学

预计学生会:

描述因素,限制和控制人口增长

显示人口周期性波动的原因

简单的解决人口问题的基础上改变少、死亡率、移民、移民

收集、展示、诠释数据

确定和描述一个金字塔能源在能源流经生态系统

展示知识的过程中继承

比较光合作用和细胞呼吸方面的算法、产品、化学方程式、负责 organelles

说明细胞呼吸和光合作用的作用

Page 144: 2001 -1995

12 年级 ( 生物 30)

一 细胞生物学 (细胞结构)

预计学生会:

描述细胞的结构和功能如下:

o 细胞膜

o 线粒体

o 粗顺利 endoplasmicreticulum o ribosomes

o Golgi 机构

o 水泡变 o vacuoles

o lysosomes

o 核信封

o 核心

o 细胞核

o 染色体

细胞间的识别功能结构

细胞结构图,确定电子毁损

B 细胞生物学 (化合物电池)

预计学生会:

水形容两极分化的结果氢原子力

说明水是溶剂的作用,调节温度、润滑

区分酸、基地、带,并指出,必须发扬对生物系统

Page 145: 2001 -1995

C 细胞生物学 (分子生物学)

预计学生会:

展示知识的综合应用和有机聚合物水解

区分碳水化合物、脂肪、蛋白质、核酸方面的化学结构

认识糖类的经验公式

区分单、双、多糖

区分淀粉、纤维素、糖原

碳水化合物的主要功能清单

与此相反,饱和脂肪和脂肪酸的分子结构

描述地点的重要性,并说明如下人体:中性脂肪、类固醇、phospholipids

以此确定普遍氨基酸及胺、硫酸(羧基)、R小组

区分初等、中等、高等教育、结构完整的蛋白质

蛋白质的主要功能清单

与一般的分子结构的自动保障作用,为"能源货币"细胞

D 细胞生物学 (DNA)

预计学生会:

四个基地的名称,并说明 DNA 用 DNA 结构如下:

o 核苷酸(糖、磷、基地)

o 找互补基地

o 双螺旋管

o 氢焊接

关于 DNA重复叙述三个基本步骤: o "unzipping"

o 找互补基地

o 与邻近 nucleotides

确定重组DNA

3重组DNA 用途说明

与此相反,一般的组成结构和 DNA核糖核酸

Page 146: 2001 -1995

e 电池及应用程序 (蛋白质合成)

预计学生会:

显示了解蛋白质合成的基本步骤,DNA 鉴定的作用,mRNA,tRNA,ribosomes笔录及翻译的过程

氨基酸顺序决定了一个特定的 DNA序列编码,由于 mRNA表 codons

举例两个环境突变可造成人体变异

用实例说明如何影响蛋白质合成的 DNA 突变,并可能导致遗传疾病

F 电池及应用程序 (癌症)

预计学生会:

对于癌症描述:

o 核心异常

o 混乱和失控的增长(anaplasia)

o 缺乏联系限制 o vascularization

o 移转

清单 7危险信号,表明可能有癌症

区分原始的基因和基因

用实例说明了发起者和推动者的角色癌发生

显示了解如何才能实现病毒癌发生

收集、展示、诠释数据

G 电池及应用程序 (交通跨越细胞膜)

预计学生会:

应用知识解释有机分子的结构和功能的变化,薄膜纸模型

Page 147: 2001 -1995

细胞膜为何被称为"地渗透"

比较和对比:传播、交通便利、渗透性、积极运输

解释因素,影响了整个细胞膜传播速度

生物学描述,包括与 phagocytosispinocytosis,与此相反,exocytosis

hypertonic 预期效果,isotonic,hypotonic 环境对动物细胞

表现出的关系和意义的认识,以面积数量、规模与范围,细胞

H 电池及应用程序 (酶)

预计学生会:

理解表现如下:新陈代谢、酶、基质,coenzyme,活化能量

为查明来源腺功能与甲状腺素、甲状腺素对代谢

解释"锁与钥匙"的行动模式 enzymatic

确定维生素的生化反应的作用

区分酶的作用和共同的生化反应酶

应用知识的解释影响酶蛋白的活动酸碱、温度、基质浓度、酶浓度、抑制剂竞争、重金属

用科学的方法设计实验

一 -人类生物学 (消化系统)

预计学生会:

并确定了各职能如下:

o 口

o 舌头

o 牙齿

o 唾液腺肿痛

o 咽头 o epiglottis

o 食管

o 心脏 sphincter

o 胃癌

Page 148: 2001 -1995

o pyloricsphincter

o 十二指肠

o 肝癌

o 胆

o 胰脏

o 小肠

o 附录

o 大肠(科隆)

o 直肠

o 肛门

以下是腺消化酶的来源,并说明这些反应促进消化:

o 唾液腺淀粉酶

o 胰腺淀粉酶

o proteases(pepsin、胰蛋白酶) o lipase

o peptidase

o Maltase

o nuclease

被形容和蠕动

鉴定部门和行动来消化医学、胰腺、肠和饮料

消息来源说,为查明腺功能胰岛素

解释的作用,全在乳化脂肪

肝功能六大名单

解剖表明正确使用显微镜

研究如何把小肠消化吸收的专门

描述功能的大肠杆菌科隆

J -人类生物学 (循环系统--血液循环和)

预计学生会:

描述和区分五种血管

识别功能,让每:

o subclavian脉络

o 颈脉

o 动脉血管

o 肠系膜动脉

o 后来发现,维纳莱 Cava

Page 149: 2001 -1995

o 肺血管

o 肝地

o 肝门精神

o 肾脏脉络

o iliac脉络

o 心脏脉络

o 主动脉

安全技术及示范正确剖析

肺循环和系统区分

确定和描述的结构不同,在胚胎和成人流通体系

展示了知识的途径,通过血液细胞的主动脉和身体回到左心室

等离子体的主要成分名单

识别功能,使淋巴微血管、纹理、节点

描述形状、功能、起源和红血细胞、白血球、血小板

显示正确使用显微镜大院

说明抗原和抗体作用

描述毛细管-组织液交流

K -人类生物学 (心血管系统--心脏结构和功能)

预计学生会:

识别功能,让每:

o 左和右 atria

o 左和右的脑室

o 心脏脉络

o 后来发现,维纳莱 Cava

o 主动脉

o 肺脉络

o 肺主干

o atrioventricular阀门 o chordaetendineae

o 半月球阀门

o 鼻腔

描述的位置和作用 sa景区、双节点、Purkinje纤维

描述了自主调节神经系统的心跳

涉及因素影响,调节血压,低血压和高血压

测量血压表现

Page 150: 2001 -1995

区分收缩压力和舒张

L -人类生物学 (呼吸系统)

预计学生会:

识别功能,让每:

o 喉癌

o 气管

o 气管 o bronchioles

o 气管

o 肋骨和膈

o 膜膜

o 胸腔

解释作用在呼吸道粘液和 cilia

解释关系的结构和功能气管

与此相反,在力学过程和吸吸

描述互动肺膜膜、排骨、膈在呼吸过程

解释作用在二氧化碳和氢离子的刺激延髓呼吸中心

描述在氧气和二氧化碳交换内外呼吸

区分运输氧和二氧化碳在血液中的作用 Oxyhemoglobin 解释,carbaminohemoglobin、血红蛋白减少、盐离子

M -人类生物学 (神经系统--神经原,产生冲动、反射局)

预计学生会:

识别功能,让每:科学家、细胞组织,Axon

区分感官、汽车、结构和功能方面 interneurons

解释传输的神经原的神经冲动通过利用下列职权:

o 休息和行动可能

o 消解和 repolarization

o 钠和钾大门

Page 151: 2001 -1995

o 钠钾泵

o 回收期

o 门槛("全或无反应")

与结构的神经纤维myelinated传导冲动的速度

鉴定的主要内容有 synapse

解释的过程中,推动跨越 synapse

展示知识 neurotransmitters 细分的连结颚

反射弧的结构是如何运作

N -人类生物学 (神经系统--分裂的大脑和神经系统)

预计学生会:

相反的,中央和地方的职能和周边神经系统

区分功能的同情和自主神经系统的交感神经师

确定并说明其来源腺的刺激作用,"打、飞行"的反应

识别功能,让每:

o 延髓

o 大脑 o thalamus

o 小脑

o 甲状腺

o 头 callosum

丘脑和垂体解释神经内分泌相互作用控制中心

O -人类生物学 (泌尿系统)

预计学生会:

识别功能,让每:

o 肾脏

o 输尿管

o 尿道

o 泌尿膀胱

Page 152: 2001 -1995

o 肾皮质

o 肾延髓

o 肾髋

识别功能,让每: o nephron

o glomerulus

o 鲍曼的物品

o Afferent 和 efferentarterioles

o 毛细管 peritubular网

o 接近和迂回不直接疼痛

o 收集管道

o 河套的 Henle

相反,在肾动脉血液和肾脏方面对尿素和葡萄糖含量

找出源头,一面为青少年保健项目,并阐释这些荷尔蒙醛固酮管制

与青少年保健项目,醛固酮,nephron钠和水的管理水平的血液

P -人类生物学 (生殖系统)

预计学生会:

识别功能,让每:

o 睾丸(seminiferous肾小管细胞和 interstitial)

o 附睾

o ductusdeferens(固网)

o 前列腺

o Cowper 的一生

o 基本水泡变

o 阴茎

o 尿道

道路上表现出知识的精子的肛门疼痛 seminiferous开幕

名单基本功能液

鉴定尾巴,midpiece团长、国家和精子成熟 acrosome职务

睾丸功能描述

显示控制知识水平的睾丸内分泌系统

并确定了各职能如下:

o 卵巢(卵泡及黄头)

o oviducts(输卵管内)

o 子宫

Page 153: 2001 -1995

o 子宫颈癌 o Vagina

o 阴蒂

雌黄激素的功能说明

描述事件的顺序、子宫卵巢周期

展示知识的控制和子宫卵巢荷尔蒙周期

知识反馈机制,表明了积极参与 oxytocin

描述激素变化,因植入

APPENDIX A: PRESCRIBED LEARNING OUTCOMESThe Prescribed Learning Outcomes for Grade 12 are listed under each of the following organizers:

A: Cell Biology (Cell Structure) B: Cell Biology (Cell Compounds) C: Cell Biology (Biological Molecules) D: Cell Biology (DNA) E: Cell Processes (Protein Synthesis) F: Cell Processes (Cancer) G: Cell Processes and Applications (Transport Across Cell Membrane) H: Cell Processes and Applications (Enzymes) I: Human Biology (Digestive System) J: Human Biology (Circulatory System - Circulation and Blood) K: Human Biology (Circulatory System - Heart Structure and Function) L: Human Biology (Respiratory System) M: Human Biology (Nervous System - Neuron, Impulse Generation, and Reflex Arc) N: Human Biology (Nervous System - Divisions of the Nervous System and the Brain) O: Human Biology (Urinary System) P: Human Biology (Reproductive System)

The Prescribed Learning Outcomes for Biology 12 have been coded for ease of reference for those compiling provincial exams.

Page 154: 2001 -1995

Cell Biology (Cell Structure)

It is expected that students will:

A1. describe the following cell structures and their functions:

cell membrane

mitochondria

smooth and rough endoplasmic reticulum

ribosomes

Golgi bodies

vesicles

vacuoles

lysosomes

nuclear envelope

nucleus

nucleolus

chromosomes

A2. identify the functional interrelationships of cell structures

A3. identify the cell structures in diagrams and electron micrographs

Cell Biology (Cell Compounds)

It is expected that students will:

B1. describe how the polarity of the water molecule results in hydrogen bonding

B2. describe the role of water as a solvent, temperature regulator, and lubricant

B3. distinguish among acids, bases, and buffers, and indicate the importance of pH to biological systems

Cell Biology (Biological Molecules)

It is expected that students will:

Page 155: 2001 -1995

C1. demonstrate a knowledge of synthesis and hydrolysis as applied to organic polymers

C2. distinguish among carbohydrates, lipids, proteins, and nucleic acids with respect to chemical structure

C3. recognize the empirical formula of a carbohydrate

C4. differentiate among monosaccharides, disaccharides, and polysaccharides

C5. differentiate among starch, cellulose, and glycogen

C6. list the main functions of carbohydrates

C7. compare and contrast saturated and unsaturated fats in terms of molecular structure

C8. describe the location and explain the importance of the following in the human body: neutral fats, steroids, phospholipids

C9. draw a generalized amino acid and identify the amine, acid (carboxyl), and R-groups

C10. differentiate among the primary, secondary, tertiary, and quaternary structure of proteins

C11. list the major functions of proteins

C12. relate the general structure of the ATP molecule to its role as the "energy currency" of cells

Cell Biology (DNA)

It is expected that students will:

D1. name the four bases in DNA and describe the structure of DNA using the following terms:

nucleotide (sugar, phosphate, base)

complementary base pairing

double helix

hydrogen bonding

Page 156: 2001 -1995

D2. describe DNA replication with reference to three basic steps:

"unzipping"

complementary base pairing

joining of adjacent nucleotides

D3. define recombinant DNA

D4. describe three uses for recombinant DNA

D5. compare and contrast the general structural composition of DNA and RNA

Cell Processes and Applications (Protein Synthesis)

It is expected that students will:

E1. demonstrate a knowledge of the basic steps of protein synthesis, identifying the roles of DNA, mRNA, tRNA, and ribosomes in the processes of transcription and translation

E2. determine the sequence of amino acids coded for by a specific DNA sequence, given a table of mRNA codons

E3. give examples of two environmental mutagens that can cause mutations in humans

E4. use examples to explain how mutations in DNA affect protein synthesis and may lead to genetic disorders

Cell Processes and Applications (Cancer)

It is expected that students will:

F1. describe cancer with respect to:

abnormal nuclei

disorganized and uncontrolled growth (anaplasia)

lack of contact inhibition

vascularization

metastasis

Page 157: 2001 -1995

F2. list the seven danger signals that may indicate the presence of cancer

F3. differentiate between a proto-oncogene and an oncogene

F4. use examples to outline the roles of initiators and promoters in carcinogenesis

F5. demonstrate a knowledge of how a virus can bring about carcinogenesis

Cell Processes and Applications (Transport Across Cell Membrane)

It is expected that students will:

G1. apply knowledge of organic molecules to explain the structure and function of the fluid-mosaic membrane model

G2. explain why the cell membrane is described as "selectively permeable"

G3. compare and contrast the following: diffusion, facilitated transport, osmosis, active transport

G4. explain factors that affect the rate of diffusion across a cell membrane

G5. describe endocytosis, including phagocytosis and pinocytosis, and contrast it with exocytosis

G6. predict the effects of hypertonic, isotonic, and hypotonic environments on animal cells

G7. demonstrate an understanding of the relationship and significance of surface area to volume, with reference to cell size

Cell Processes and Applications (Enzymes)

It is expected that students will:

Page 158: 2001 -1995

H1. demonstrate an understanding of the following terms: metabolism, enzyme, substrate, coenzyme, activation energy

H2. identify the source gland for thyroxin and relate the function of thyroxin to metabolism

H3. explain the "lock and key" model of enzymatic action

H4. identify the role of vitamins in biochemical reactions

H5. differentiate between the roles of enzymes and co-enzymes in biochemical reactions

H6. apply knowledge of proteins to explain the effects on enzyme activity of pH, temperature, substrate concentration, enzyme concentration, competitive inhibitors, and heavy metals

H7. devise an experiment using the scientific method

Human Biology (Digestive System)

It is expected that students will:

I1. identify and give a function for each of the following:

mouth

tongue

teeth

salivary glands

pharynx

epiglottis

esophagus

cardiac sphincter

stomach

pyloric sphincter

duodenum

liver

gall bladder

pancreas

small intestine

appendix

large intestine (colon)

rectum

Page 159: 2001 -1995

anus

I2. relate the following digestive enzymes to their glandular sources and describe the digestive

reactions they promote:

salivary amylase

pancreatic amylase

proteases (pepsin, trypsin)

lipase

peptidase

maltase

nuclease

I3. describe swallowing and peristalsis

I4. identify the components and describe the digestive actions of gastric, pancreatic, and intestinal juices

I5. identify the source gland for and describe the function of insulin

I6. explain the role of bile in the emulsification of fats

I7. list six major functions of the liver

I8. demonstrate the correct use of the dissection microscope

I9. examine the small intestine and describe how it is specialized for digestion and absorption

I10. describe the functions of E. coli in the colon

Human Biology (Circulatory System--Circulation and Blood)

It is expected that students will:

J1. describe and differentiate among the five types of blood vessels

J2. identify and give functions for each of the following:

subclavian arteries and veins

jugular veins

carotid arteries

mesenteric arteries

anterior and posterior vena cava

Page 160: 2001 -1995

pulmonary veins and arteries

hepatic vein

hepatic portal vein

renal arteries and veins

iliac arteries and veins

coronary arteries and veins

aorta

J3. demonstrate safe and correct dissection techniques

J4. distinguish between pulmonary and systemic circulation

J5. identify and describe differences in structure and circulation between fetal and adult systems

J6. demonstrate a knowledge of the path of a blood cell from the aorta through the body and back to the left ventricle

J7. list the major components of plasma

J8. identify and give functions of lymph capillaries, veins, and nodes

J9. describe the shape, function, and origin of red blood cells, white blood cells, and platelets

J10. demonstrate the correct use of the compound microscope

J11. explain the roles of antigens and antibodies

J12. describe capillary-tissue fluid exchange

Human Biology (Circulatory System--Heart Structure and Function)

It is expected that students will:

K1. identify and give functions for each of the following:

left and right atria

left and right ventricles

coronary arteries and veins

anterior and posterior vena cava

aorta

Page 161: 2001 -1995

pulmonary arteries and veins

pulmonary trunk

atrioventricular valves

chordae tendineae

semi-lunar valves

septum

K2. describe the location and functions of the SA node, AV node, and Purkinje fibres

K3. describe the autonomic regulation of the heartbeat by the nervous system

K4. relate factors that affect and regulate blood pressure to hypertension and hypotension

K5. demonstrate the measurement of blood pressure

K6. distinguish between systolic and diastolic pressures

Human Biology (Respiratory System)

It is expected that students will:

L1. identify and give functions for each of the following:

larynx

trachea

bronchi

bronchioles

alveoli

diaphragm and ribs

pleural membranes

thoracic cavity

L2. explain the roles of cilia and mucus in the respiratory tract

L3. explain the relationship between the structure and function of alveoli

L4. compare and contrast the mechanics of the processes of inhalation and exhalation

Page 162: 2001 -1995

L5. describe the interaction of the lungs, pleural membranes, ribs, and diaphragm in the breathing process

L6. explain the roles of carbon dioxide and hydrogen ions in stimulating the breathing centre in the medulla oblongata

L7. describe the exchange of carbon dioxide and oxygen during internal and external respiration

L8. distinguish between the transport of in the blood by explaining the roles of oxyhemoglobin, carbaminohemoglobin, reduced hemoglobin, and bicarbonate ions

Human Biology (Nervous System--Neuron, Impulse Generation, and Reflex Arc)

It is expected that students will:

M1. identify and give functions for each of the following: dendrite, cell body, axon

M2. distinguish among sensory, motor, and interneurons with respect to structure and function

M3. explain the transmission of a nerve impulse through a neuron, using the following terms:

resting and action potential

depolarization and repolarization

sodium and potassium gates

sodium-potassium pump

recovery period

threshold ("all-or-none response")

M4. relate the structure of a myelinated nerve fibre to the speed of impulse conduction

M5. identify the major components of a synapse

M6. explain the process by which impulses travel across a synapse

M7. demonstrate knowledge of how neurotransmitters are broken down in the synaptic cleft

Page 163: 2001 -1995

M8. relate the structure of a reflex arc to how it functions

Human Biology (Nervous System--Divisions of the Nervous System and the Brain)

It is expected that students will:

N1. contrast the locations and functions of the central and peripheral nervous systems

N2. differentiate between the functions of the sympathetic and parasympathetic divisions of the autonomic nervous system

N3. identify the source gland for adrenalin and explain its role in the "fight or flight" response

N4. identify and give functions for each of the following:

medulla oblongata

cerebrum

thalamus

cerebellum

hypothalamus

corpus callosum

N5. explain how the hypothalamus and pituitary gland interact as the neuroendocrine control

centre

Human Biology (Urinary System)

It is expected that students will:

O1. identify and give functions for each of the following:

kidney

ureter

urethra

urinary bladder

renal cortex

renal medulla

renal pelvis

Page 164: 2001 -1995

O2. identify and give functions for each of the following:

nephron

glomerulus

Bowman's capsule

afferent and efferent arterioles

peritubular capillary network

proximal and distal convoluted tubules

collecting duct

loop of Henle

O3. contrast the blood in the renal artery and the renal vein with respect to urea and glucose

content

O4. identify the source glands for ADH and aldosterone and explain how these hormones are regulated

O5. relate ADH, aldosterone, and the nephron to the regulation of water and sodium levels in the blood

Human Biology (Reproductive System)

It is expected that students will:

P1. identify and give functions for each of the following:

testes (seminiferous tubules and interstitial cells)

epididymis

ductus (vas) deferens

prostate gland

Cowper's glands

seminal vesicles

penis

urethra

P2. demonstrate a knowledge of the path of sperm from the seminiferous tubules to the urethral

opening

P3. list the functions of seminal fluid

P4. identify the tail, midpiece, head, and acrosome of a mature sperm and state their functions

P5. describe the functions of testosterone

Page 165: 2001 -1995

P6. demonstrate a knowledge of the control of testosterone levels by the endocrine system

P7. identify and give a function for each of the following:

ovaries (follicles and corpus luteum)

oviducts (fallopian tubes)

uterus

cervix

vagina

clitoris

P8. describe the functions of estrogen

P9. describe the sequence of events in the ovarian and uterine cycles

P10. demonstrate knowledge of the control of the ovarian and uterine cycles by hormones

P11. demonstrate knowledge of a positive feedback mechanism involving oxytocin

P12. describe the hormonal changes that occur as a result of implantation

Table of Contents

Province of British ColumbiaMinistry of EducationCurriculum Branch ?1996 Copyright

Maintained by: Biology Coordinator

Updated: July 14, 2003

Page 166: 2001 -1995

Ministry of Education Home Page

What is Appendix B?

Appendix B consists of general information on learning resources, as well as Grade Collection

information and alphabetical annotations of the provincially recommended resources.

What is a Grade Collection?

A Grade Collection is the format used to organize the provincially recommended learning

resources by grade and by curriculum organizer. It can be regarded as a "starter set" of basic

resources to deliver the curriculum. In many cases, the Grade Collection provides a choice of

more than one resource to support curriculum organizers, enabling teachers to select

resources that best suit different teaching and learning styles.

There may be prescribed learning outcomes either partially or not at all supported by learning

resources at this time. Many of these are best met by teacher-developed activities. Teachers

may also wish to supplement Grade Collection resources with locally selected materials.

What kinds of resources are found in a Grade Collection?

Learning resources in a Grade Collection are categorized as either comprehensive or

additional. Comprehensive resources provide a broad coverage of the learning outcomes for

most curriculum organizers. Additional resources are more topic specific and support

individual curriculum organizers or clusters of outcomes. They provide valuable support for or

extension to specific topics and are typically used to supplement or fill in the areas not

covered by the comprehensive resources.

How are Grade Collections kept current?

Under the provincial continuous submissions process, suppliers advise the ministry about

newly developed resources as soon as they are released. Resources judged to have a

potentially significant match to the learning outcomes for individual IRPs are evaluated by

practising classroom teachers who are trained by ministry staff to use provincial evaluation

criteria. Resources selected for provincial recommendation receive Ministerial Order and are

added to the existing Grade Collections. The ministry updates the Grade Collections on a

regular basis on the ministry web site at

http://www.bced.gov.bc.ca/irp_resources/lr/resource/gradcoll.htm. Please check this site for

the most current and up-to-date version of Appendix B.

How long do learning resources keep their recommended status?

Page 167: 2001 -1995

Learning resources will retain their recommended status for a minimum of five years after

which time they may be withdrawn from the Grade Collections, thereby terminating their

provincially recommended status. Decisions regarding the withdrawal of learning resources

will be based on, but not limited to, considerations of curriculum support, currency, and

availability. Schools may continue to use a learning resource after withdrawal provided local

school board approval is obtained.

How can teachers choose learning resources to meet their classroom needs?

As outlined in Evaluating, Selecting and Managing Learning Resources: A Guide (Revised

2000), there are a number of approaches to selecting learning resources.

Teachers may choose to use:

provincially recommended resources to support provincial or locally developed

curricula

resources that are not on the ministry's provincially recommended list (resources that

are not on the provincially recommended list must be evaluated through a local,

board-approved process).

The Ministry of Education has developed a variety of tools and guidelines to assist teachers

with the selection of learning resources. These include:

Evaluating, Selecting and Managing Learning Resources: A Guide (Revised 2000)

with accompanying CD-ROM tutorial and evaluation instruments

Grade Collection(s) in each IRP. Each Grade Collection begins with a chart which

lists both comprehensive and additional resources for each curriculum organizer. The

chart is followed by an annotated bibliography with supplier and ordering information.

(Price and supplier information should be confirmed at the time of ordering). There is

also a chart that lists Grade Collection titles alphabetically and a blank planning

template that can be used by teachers to record their individual choices

Resource databases on CD-ROM or on-line

Sets of recommended learning resources are available in a number of host districts

throughout the province to allow teachers to examine the materials first hand at

regional displays.

Catalogue of Recommended Learning Resources

What are the criteria to consider when selecting learning resources?

There are a number of factors to consider when selecting learning resources.

Content

The foremost consideration for selection is the curriculum to be taught. Prospective resources

must adequately support the particular learning objectives that the teacher wants to address.

Page 168: 2001 -1995

Teachers will determine whether a resource will effectively support any given learning

outcomes within a curriculum organizer. This can only be done by examining descriptive

information regarding that resource; acquiring additional information about the material from

the supplier, published reviews, or colleagues; and by examining the resource first-hand.

Instructional Design

When selecting learning resources, teachers must keep in mind the individual learning styles

and abilities of their students, as well as anticipate the students they may have in the future.

Resources should support a variety of special audiences, including gifted, learning disabled,

mildly intellectually disabled, and ESL students. The instructional design of a resource

includes the organization and presentation techniques; the methods used to introduce,

develop, and summarize concepts; and the vocabulary level. The suitability of all of these

should be considered for the intended audience.

Teachers should also consider their own teaching styles and select resources that will

complement them. The list of recommended resources contains materials that range from

prescriptive or self-contained resources, to open-ended resources that require considerable

teacher preparation. There are recommended materials for teachers with varying levels and

experience with a particular subject, as well as those that strongly support particular teaching

styles.

Technical Design

While the instructional design of a package will determine the conceptual organization, it is

the technical design that brings that structure into reality. Good technical design enhances

student access and understanding. Poor technical quality creates barriers to learning.

Teachers should consider the quality of photographs and illustrations, font size and page

layout, and durability. In the case of video, audible and age appropriate narration and

variation in presentation style should be considered. When selecting digital resources,

interactivity, feedback, constructive engagement, usability, and functionality are important.

Social Considerations

An examination of a resource for social considerations helps to identify potentially

controversial or offensive elements which may exist in the content or presentation. Such a

review also highlights where resources might support pro-social attitudes and promote

diversity and human rights issues.

The intent of any Social Considerations screening process, be it at the local or provincial

level, is not to remove controversy, but to ensure that controversial views and opinions are

presented in a contextual framework.

Page 169: 2001 -1995

All resources on the ministry's recommended list have been thoroughly screened for social

concerns from a provincial perspective. However, teachers must consider the

appropriateness of any resource from the perspective of the local community.

Media

When selecting resources, teachers should consider the advantages of various media. Some

topics may be best taught using a specific medium. For example, video may be the most

appropriate medium when teaching a particular, observable skill, since it provides a visual

model that can be played over and over or viewed in slow motion for detailed analysis. Video

can also bring otherwise unavailable experiences into the classroom and reveal "unseen

worlds" to students. Software may be particularly useful when students are expected to

develop critical-thinking skills through the manipulation of a simulation, or where safety or

repetition are factors. Print or CD-ROM resources can best be used to provide extensive

background information on a given topic. Once again, teachers must consider the needs of

their individual students, some of whom may learn better from the use of one medium than

another.

Use of Information Technology

Teachers are encouraged to embrace a variety of educational technologies in their

classrooms. To do so, they will need to ensure the availability of the necessary equipment

and familiarize themselves with its operation. If the equipment is not currently available, then

the need must be incorporated into the school or district technology plan.

What funding is available for purchasing learning resources?

As part of the selection process, teachers should be aware of school and district funding

policies and procedures to determine how much money is available for their needs. Funding

for various purposes, including the purchase of learning resources, is provided to school

districts.

Learning resource selection should be viewed as an ongoing process that requires a

determination of needs, as well as long-term planning to co-ordinate individual goals and local

priorities.

Existing Materials

Prior to selecting and purchasing new learning resources, an inventory of those resources

that are already available should be established through consultation with the school and

district resource centres. In some districts, this can be facilitated through the use of district

and school resource management and tracking systems. Such systems usually involve a

computer database program (and possibly bar-coding) to help keep track of a multitude of

Page 170: 2001 -1995

titles. If such a system is put on-line, then teachers can check the availability of a particular

resource via computer.